Sei sulla pagina 1di 48

FÍSICA Professor Gomes

Nota de Aula
CAPÍTULO

                           

Neste Capítulo
1
1 Introdução
2 Temperatura e Equilíbrio
térmico
3 Termômetros e escalas de
temperatura
4 Termômetro de gás e escala
Kelvin
5 Expansão térmica
6 Quantidade de calor
7 Calorimetria e transições de
fases
8 Mecanismos de transferência
de calor
 
 
 
 
 

Temperatura e Calor

2018 www.professorgomes.com.br
 

NOTA DE AULA 
PROF. JOSÉ GOMES RIBEIRO FILHO 
 
TEMPERATURA E CALOR
 
1 INTRODUÇÃO 
Tanto  em  um  dia  escaldante  de  verão  quanto  em  uma  noite  fria  de  inverno,  seu  corpo  precisa  manter  uma 
temperatura aproximadamente constante. Ele possui mecanismos de controle de temperatura eficientes, mas, algumas 
vezes, precisa de ajuda. Em um dia quente, você usa menos roupa para melhorar a troca de calor entre seu corpo e o ar 
ambiente, e para melhorar o resfriamento produzido pela evaporação do suor. Você toma bebidas geladas e talvez fique 
em uma sala com ar‐condicionado, ou perto de um ventilador. Em um dia frio, você usa mais roupas ou fica dentro de 
casa em local quente. Quando você sai de casa, procura manter‐se ativo e bebe líquidos quentes para ficar aquecido. Os 
conceitos deste capítulo auxiliarão você a entender os processos físicos básicos para preservar o calor ou o frio.  
Os termos ‘temperatura’ e ‘calor’ costumam ser usados como sinônimos na linguagem do dia‐a‐dia. Em física, 
contudo, esses dois termos têm significados bastante diferentes. Neste capítulo, definiremos temperatura em termos 
de  sua  medição,  e  veremos  como  a  variação  de  temperatura  afeta  as  dimensões  dos  objetos.  Veremos  que  calor  se 
refere  à  transferência  de  energia  provocada  pelas  diferenças  de  temperatura,  e  aprenderemos  a  calcular  e  controlar 
essas transferências de energia.  
Daremos  ênfase  neste  capítulo  aos  conceitos  de  temperatura  e  de  calor  em  suas  relações  com  objetos 
macroscópicos, tais como cilindros de gás, cubos de gelo e o corpo humano. Nos próximos capítulos estudaremos esses 
mesmos conceitos sob o ponto de vista microscópico, referente ao comportamento dos átomos e moléculas do sistema. 
Esses capítulos iniciais fornecerão ferramentas básicas para a termodinâmica, o estudo das transformações de energia 
envolvendo calor, trabalho mecânico e outros tipos de energia, e de como essas transformações se relacionam com as 
propriedades da matéria. A termodinâmica constitui uma parte indispensável dos fundamentos da física, da química e 
da  biologia,  e  encontra  aplicação  em  áreas  como  motores  de  automóveis,  refrigeradores,  processos  bioquímicos  e  a 
estrutura das estrelas.  
 
2 TEMPERATURA E EQUILÍBRIO TÉRMICO 
O conceito de temperatura tem origem nas ideias qualitativas de ‘quente’ e ‘frio’, que são baseadas em nosso 
tato.  Um  corpo  que  parece  estar  quente  normalmente  está  em  uma  temperatura  mais  elevada  do  que  um  corpo 
análogo  que  parece  estar  frio.  Isso  é  vago,  e  os  sentidos  podem  ser  enganosos.  Contudo,  muitas  propriedades  da 
matéria que podemos medir dependem da temperatura. O comprimento de uma barra metálica, a pressão no interior 
de uma caldeira, a intensidade da corrente elétrica transportada por um fio e a cor de um objeto incandescente muito 
quente — todas essas grandezas dependem da temperatura. 
A temperatura está relacionada à energia cinética das moléculas de um material. Simplificadamente, podemos 
conceituar  a  grandeza  temperatura  como  uma  medida  que  indica  o  grau  de  agitação  térmica  das  partículas  de  um 
sistema. Em geral, essa relação é bastante complexa. No próximo capítulo, vamos estudar a relação entre a temperatura 
e  a  energia  do  movimento  das  moléculas  de  um  gás  ideal.  Entretanto,  é  importante  entender  que  o  calor  e  a 
temperatura  podem  ser  definidos  independentemente  de  qualquer  movimento  molecular.  A  princípio,  vamos 
desenvolver uma definição macroscópica de temperatura. 
Antes  de  usar  a  temperatura  como  uma  medida  para  saber  se  um  corpo  está  quente  ou  frio,  precisamos 
construir uma escala de temperatura. Para isso, podemos usar qualquer propriedade do sistema que dependa do fato 
de o corpo estar ‘quente’ ou ‘frio’. A figura 1a mostra um conhecido sistema para medir temperatura. Quando o sistema 
torna‐se mais quente, um líquido (geralmente o etanol ou o mercúrio) se expande e sobe no tubo, e o valor de L cresce. 
Outro  sistema  simples  é  um  gás  no  interior  de  um  recipiente  mantido  a  volume  constante  (figura  1b).  A  pressão  p, 
medida  com  o  manômetro,  aumenta  ou  diminui  à  medida  que  o  gás  se  aquece  ou  esfria.  Um  terceiro  exemplo  é  a 
resistência elétrica R de um fio condutor, a qual varia quando o fio se aquece ou esfria. Cada uma dessas propriedades 
nos  fornece  um  número  (L,  p  ou  R)  que  varia  quando  o  corpo  se  aquece  ou  esfria,  de  modo  que  a  respectiva 
propriedade pode ser usada para fazer um termômetro. 

 

 
 
 
 
 
FIGURA  1  Dois  instrumentos  para  medir  a 
temperatura. 
 

 
Para  medir  a  temperatura  de  um  corpo,  você  coloca  o  termômetro  em  contato  com  o  corpo.  Se  você  deseja 
saber a temperatura de uma xícara com café quente, coloca o bulbo do termômetro no café; quando ele interage com o 
líquido, o termômetro se aquece e o café esfria ligeiramente. Quando o estado estacionário é atingido, você pode ler a 
temperatura.  Dizemos  que  o  sistema  atingiu  o  equilíbrio,  um  estado  em  que  não  existe  mais  nenhuma  variação  de 
temperatura nem do termômetro nem do café. Chamamos esse estado de equilíbrio térmico.  
Quando dois sistemas estão separados por um material isolante, tal como a madeira, o plástico, o isopor ou a 
fibra  de  vidro,  um  sistema  influencia  o  outro  muito  lentamente.  As  geladeiras  usadas  em  piqueniques  são  feitas com 
materiais isolantes para impedir que o gelo e os alimentos gelados se aqueçam e atinjam o equilíbrio térmico com o ar 
quente  do  verão  fora  da  geladeira.  Um  isolante  ideal  é  um  material  que  impede  qualquer  tipo  de  interação  entre  os 
sistemas. Ele impede que o equilíbrio térmico seja atingido quando os dois sistemas não estão em equilíbrio no início. 
Um isolante ideal é, como o próprio nome indica, uma idealização; um isolante real, como as geladeiras de piquenique, 
não é ideal, de modo que o conteúdo da geladeira acabará esquentando. 
A LEI ZERO DA TERMODINÂMICA 
Podemos descobrir uma propriedade importante do equilíbrio térmico considerando três sistemas A, B e C, que 
não estão inicialmente em equilíbrio térmico (figura 2). Colocamos os sistemas no interior de uma caixa isolante ideal 
para que não possam interagir com nada a não ser um com o outro. Separamos A e B por meio de uma parede isolante 
ideal (figura 2a), porém, deixamos C interagir com A e com B. Essa interação ocorre porque as paredes entre C e A e 
entre C e B são constituídas por um material condutor térmico, um material que permite a interação térmica através 
dele.  Esperamos  até  que  o  equilíbrio  térmico  seja  atingido;  então  A  e B  estão  simultaneamente  em  equilíbrio  com  C. 
Porém, será que o sistema A está em equilíbrio térmico com o sistema B?  
Para  responder  a  essa  pergunta,  separamos  o  sistema  C  de  A  e  de  B  por  meio  de  uma  parede  isolante  ideal 
(figura  2b)  e,  a  seguir,  trocamos  a  parede  isolante  que  existia  entre  eles  por  uma  parede  condutora  que  permite  a 
interação entre A e B. O que ocorrerá? A experiência mostra que nada ocorrerá; não haverá nenhuma interação entre A 
e B. Concluímos, então, que: 
Quando  C  está  em  equilíbrio  térmico  com  A  e  com  B,  então  A  também  está  em  equilíbrio  com  B.  Esse  fenômeno  é 
conhecido como a lei zero da termodinâmica. 
(A importância dessa lei só foi reconhecida depois que a primeira, a segunda e a terceira leis foram enunciadas. Como 
essa lei é básica para as demais leis, o nome ‘lei zero’ parece apropriado.)  
Suponha agora que o sistema C seja um termômetro, tal como o termômetro de bulbo com líquido da figura 1a. 
Na figura 2a, o termômetro C está em contato com A e com B. No equilíbrio térmico, quando a leitura do termômetro 
atingir  um  valor  estável,  ele  estará  medindo  a  temperatura  tanto  de  A  quanto  de  B;  logo,  A  e  B  possuem  a  mesma 
temperatura.  
 
 
 
 
 
FIGURA 2 A lei zero da termodinâmica. 
 

 
 

A  experiência  mostra  que  o  equilíbrio  térmico  não  é  alterado  quando  se  introduz  ou  se  remove  um  isolante; 
logo,  a  leitura  do  termômetro  C  não  se  alteraria  se  ele  estivesse  em  contato  separadamente  com  A  ou  com  B. 
Concluímos, assim, que: 
Dois sistemas estão em equilíbrio térmico se e somente se eles possuem a mesma temperatura. 
É isso que torna útil o termômetro; na realidade, um termômetro mede sua própria temperatura, mas quando 
um  termômetro  está  em  equilíbrio  térmico  com  outro  corpo,  as  temperaturas  devem  ser  iguais.  Quando  as 
temperaturas de dois sistemas são diferentes, eles não podem estar em equilíbrio térmico. 
 
3 TERMÔMETROS E ESCALAS DE TEMPERATURA 
Para  que  o  dispositivo  com  líquido  no  bulbo  mostrado  na  figura  1a  se  transforme  em  um  termômetro  útil,  é 
necessário marcar uma escala numérica sobre o vidro. Esses números são arbitrários, e historicamente muitos sistemas 
diferentes têm sido utilizados. Suponha que o ‘zero’ da escala corresponda ao ponto de congelamento da água pura e o 
número  ‘100’  corresponda  ao  ponto  de  ebulição,  e  a  distância  entre  essas  duas  marcações  seja  subdividida  em  100 
intervalos  iguais  chamados  de  graus.  Isso  corresponde  à  escala  Celsius  de  temperatura  (também  chamada  de  escala 
centígrada). A temperatura Celsius é um número negativo quando se refere a um estado cuja temperatura é menor do 
que a do ponto de congelamento da água. A escala Celsius é usada na vida cotidiana, na ciência e na indústria em quase 
todos os países do mundo.  
Outro tipo de termômetro utiliza uma lâmina bimetálica, obtida com a junção de dois metais diferentes (figura 
3a). Quando a temperatura desse sistema aumenta, um dos metais se dilata mais do que o outro, e a lâmina composta 
se  encurva  (figura  3b).  Essa  lâmina  costuma  ser  enrolada  em  espiral,  com  a  extremidade  externa  fixa  na  caixa  do 
termômetro  e  a  extremidade  interna  ligada  a  um  ponteiro  (figura  3c).  O  ponteiro  gira  em  reação  à  variação  de 
temperatura. 

 
FIGURA 3 Lâmina bimetálica funcionando como termômetro. 
 
Em  um  termômetro  de  resistência,  a  variação  de  temperatura  pode  ser  medida  pela  variação  do  valor  da 
resistência elétrica de um fio fino, de cilindro de carbono ou cristal de germânio. Como a resistência pode ser medida 
com grande precisão, os termômetros de resistência, em geral, são mais precisos do que os outros tipos de termômetro.  
Alguns  termômetros  funcionam  detectando  a  quantidade  de  radiações  infravermelhas  emitidas  por  um  objeto.    Um 
exemplo moderno é o termômetro de testa ou termômetro infravermelho (figura 4). Uma enfermeira passa um desses 
termômetros sobre a testa do paciente nas proximidades da artéria temporal, e um sensor de radiações infravermelhas 
no termômetro mede a radiação que vem da pele. Os testes mostram que esse aparelho fornece valores mais precisos 
da temperatura corporal do que outros termômetros.  
 
 
 
FIGURA  4  Um  termômetro  de  testa,  ou  termômetro 
infravermelho,  mede  a  radiação  infravermelha  da  pele  que 
recobre uma das mais importantes artérias da cabeça, embora o 
revestimento  do  termômetro  toque  a  pele,  o  detector  de 
infravermelho dentro do termômetro não toca. 
 

 
 

Na escala Fahrenheit de temperatura, ainda bastante usada cotidianamente em países como os Estados Unidos, 
a temperatura de congelamento da água é 32 °F (trinta e dois graus Fahrenheit), e a temperatura de ebulição é 212 °F, 
ambas  em  condições  normais  de  pressão  atmosférica.  Há  180  graus  entre  a  temperatura  de  congelamento  e  a  de 
ebulição, em vez dos 100 graus da escala Celsius, portanto um grau Fahrenheit corresponde a apenas 100/180 ou 5/9 de 
um grau na escala Celsius.  
Indicando por TF a temperatura expressa na escala Fahrenheit e por TC a mesma temperatura expressa na escala 
Celsius, podemos estabelecer a correspondência entre esses valores comparando dois termômetros graduados nessas 
escalas como na figura 5.  
 
 
 
 
 
 
FIGURA 5 Representação das escalas Fahrenheit (A) e Celsius (B). 
 

 
Em símbolos: 
TF  32 T 0 T  32 TC  0
 C  F 
212  32 100  0 180 100  
então
TF  32 TC
             [1] 
9 5
Recomendamos  que  você  não  memorize  a  equação  (1).  Em  vez  disso,  tente  entender  o  raciocínio  usado  e 
deduza novamente essa relação quando você precisar dela. 
 
4 TERMÔMETRO DE GÁS E ESCALA KELVIN 
Quando calibramos dois termômetros — por exemplo, um termômetro com líquido no interior de um bulbo e 
um  termômetro  de  resistência  —,  fazendo  as  duas  leituras  coincidirem  em  0  °C  e  em  100  °C,  as  leituras  podem  não 
coincidir precisamente nas temperaturas intermediárias. Qualquer escala de temperatura definida desse modo sempre 
depende em parte das propriedades específicas dos materiais usados. Idealmente, seria preciso definir uma escala de 
temperaturas  que  não  dependesse  das  propriedades  de  um  material  particular.  Para  isso,  são  necessários  alguns 
princípios  da  termodinâmica.  Voltaremos  a  essa  questão  fundamental  em  um  capítulo  posterior.  Aqui  discutiremos  o 
termômetro de gás, um tipo de termômetro que apresenta um comportamento próximo do ideal.  
O  termômetro  de  gás  se  baseia  no  fato  de  que  a  pressão  de  um  gás  mantido  a  volume  constante  aumenta 
quando a temperatura aumenta. Um gás é colocado no interior de um recipiente mantido a volume constante (figura 
6a), e sua pressão é medida. Para calibrar um termômetro de gás a volume constante, medimos as pressões em duas 
temperaturas diferentes, digamos 0 °C e 100 °C, assinalamos esses pontos sobre um gráfico e desenhamos uma linha 
reta, ligando‐os. Podemos então usar esse gráfico para ler a temperatura correspondente a qualquer outra pressão. A 
figura 6b mostra os resultados de três experiências desse tipo, cada uma usando um tipo e uma quantidade diferente de 
gás.  
Extrapolando  esse  gráfico,  vemos  que  deve  existir  uma  temperatura  hipotética  igual  a  ‐273,15  °C,  em  que  a 
pressão absoluta do gás deveria ser igual a zero. Você poderia pensar que essa temperatura seria diferente para gases 
diferentes,  contudo  verifica‐se  que  ela  é  sempre  a  mesma  para  qualquer  tipo  de  gás  (pelo  menos  no  limite  de 
densidades  muito  pequenas).  Na  verdade,  é  impossível  observar  esse  ponto  de  pressão  igual  a  zero.  Os  gases  se 
liquefazem e depois se solidificam à medida que a temperatura atinge valores muito pequenos, e a proporcionalidade 
entre a pressão e a temperatura deixa de ser válida. 
Usamos  essa  temperatura  extrapolada  para  uma  pressão  nula  como a  base  para  definir  uma  escala  cujo  zero 
corresponde  a  essa  temperatura.  Essa  escala  denomina‐se  escala  Kelvin  de  temperatura,  assim  chamada  em 
homenagem ao físico inglês Lord Kelvin (1824‐1907). As unidades dessa escala são as mesmas que as da escala Celsius, 
porém o zero é deslocado de tal modo que 0 K = ‐273,15 °C e 273,15 K = 0 °C, ou seja, 
TK =TC + 273,15          [2] 
 

Essa escala é demonstrada na figura 6b. Para uma temperatura ambiente de 20 °C, obtemos 20 + 273,15, ou cerca de 
293 K. 

 
FIGURA 6 (a) Um termômetro de gás a volume constante. (b) Quanto maior a quantidade de gás no termômetro, mais 
alto é o gráfico da pressão P em função da temperatura T. 
 
A ESCALA KELVIN E A TEMPERATURA ABSOLUTA 
A escala Celsius tem dois pontos fixos: o ponto de congelamento normal da água e o ponto de ebulição da água. 
Podemos, no entanto, definir a escala Kelvin usando um termômetro de gás com apenas um ponto de referência para a 
temperatura. Definimos a razão entre duas temperaturas T1 e T2 na escala Kelvin como a razão entre as pressões P1 e P2 
medidas pelo termômetro de gás:  
T2 P2
             [3] 
T1 P1
A pressão P é diretamente proporcional à temperatura na escala Kelvin, conforme mostra a figura 6b. Para completar a 
definição  de  T,  basta  especificar  a  temperatura  Kelvin  de  um  único  estado  específico.  Por  razões  de  precisão  e  de 
facilidade de reprodução das condições, o ponto escolhido é o ponto triplo da água. Esse é o único ponto em que a água 
sólida (gelo), a água líquida e o vapor d’água podem coexistir em equilíbrio. Isso ocorre a uma temperatura de 0,01 °C e 
uma pressão de vapor igual a 610 Pa (cerca de 0,006 atm). (Essa pressão é da água; não tem nenhuma relação com a 
temperatura no gás do termômetro.) A temperatura do ponto triplo da água Ttriplo é definida pelo valor Ttriplo = 273,16 K, 
correspondente a 0,01 °C. Pela equação (3), se Ptriplo for a pressão em um  termômetro de gás para uma temperatura 
Ttriplo e P for a pressão para uma outra temperatura T, então T é dada na escala Kelvin por 
P P
T  TTriplo  (273,16K)         [4] 
PTriplo PTriplo
Verifica‐se que os termômetros contendo gases a baixas pressões são bastante precisos, mas como eles ocupam 
volumes  muito  grandes,  levam  muito  tempo  para  atingir  o  equilíbrio  térmico.  Eles  são  usados  principalmente  para 
estabelecer  padrões  com  elevada  precisão  e  para  calibrar  outros  termômetros.  As  relações  entre  as  três  escalas  de 
temperatura que discutimos são apresentadas graficamente na figura 7. 
 
 
FIGURA 7 Relações entre as escalas Kelvin (K), Celsius (C) e 
Fahrenheit  (F).  As  frações  dos  graus  das  temperaturas 
foram aproximadas para os graus inteiros mais próximos. 
 

 

A  escala  Kelvin  denomina‐se  escala  absoluta  de  temperatura,  e  seu  ponto  zero  (T  =  0  K  =  –273,15  °C,  a 
temperatura  que  na  equação  (4)  corresponde  a  P  =  0)  denomina‐se  zero  absoluto.  No  zero  absoluto,  um  sistema 
molecular (tal como uma porção de um gás, de um líquido ou de um sólido) possui um valor mínimo para a energia total 
(energia  cinética  mais  energia  potencial);  contudo,  por  causa  de  efeitos  quânticos,  não  é  correto  dizer  que  todo 
movimento  molecular  cessa  no  zero  absoluto.  Para  definir  de  modo  mais  preciso  o  que  significa  o  zero  absoluto, 
precisamos usar os princípios termodinâmicos que serão desenvolvidos nos capítulos que se seguem.  
 
5 EXPANSÃO TÉRMICA  
A  maioria  dos  materiais  sofre  expansão  ou  dilatação  térmica  quando  aquecidos.  Temperaturas  em  elevação 
fazem  o  líquido  se  expandir  em  um  termômetro  formado  por  um  líquido  dentro  de  um  tubo  (figura  1a)  e  curvam 
lâminas  bimetálicas  (figura  3b).  As  estruturas  das  pontes  devem  ser  projetadas  com  suportes  e  juntas  especiais  para 
permitir a dilatação dos materiais. Uma garrafa cheia de água e tampada muito firmemente pode quebrar quando for 
aquecida;  no  entanto,  você  pode  afrouxar  a  tampa  metálica  de  um  recipiente  jogando  água  quente  sobre  ela.  Todas 
essas situações exemplificam a dilatação térmica. 
DILATAÇÃO LINEAR  
Suponha que uma barra possua comprimento L0 em uma dada temperatura T0. Quando a temperatura varia de 
ΔT,  o  comprimento  varia  de  ΔL.  A  experiência  mostra  que,  quando  ΔT  não  é  muito  grande  (digamos,  menor  do  que 
cerca  de  100  °C),  ΔL  é  diretamente  proporcional  a  ΔT  (figura  8a).  Quando  duas  barras  feitas  com  o  mesmo  material 
sofrem  a  mesma  variação  de  temperatura,  mas  uma  possui  o  dobro  do  comprimento  da  outra,  então  a  variação  do 
comprimento também é duas vezes maior.  Portanto, ΔL também deve ser proporcional a L0 (figura 8b).  
 
 
 
 
FIGURA  8  Como  o 
comprimento  de  uma  barra 
se  comporta  com  uma 
variação  na  temperatura. 
(As  variações  de 
comprimento  são 
exageradas  para  maior 
visibilidade.) 
 

 
Introduzindo  uma  constante  de  proporcionalidade  (que  não  é  a  mesma  para  todos  os  materiais),  podemos 
expressar essas dependências mediante a equação:  
ΔL = L0αΔT (dilatação térmica linear)       [6]  
Se o comprimento de um corpo a uma temperatura T0 é L0, então seu comprimento L a uma temperatura T = T0 
+ ΔT é 
L = L0 + ΔL = L0 + αL0 ΔT = L0(1 + αΔT)       [7] 
A  constante  α,  que  descreve  as  propriedades  de  expansão  térmica  de  um  dado  material,  denomina‐se 
coeficiente de dilatação linear. As unidades de α são K–1 ou (°C)−1. (Lembre‐se de que o intervalo de um grau é o mesmo 
na  escala  Kelvin  e  na  escala  Celsius.)  Em  muitos  materiais,  as  dimensões  lineares  sofrem  variações  de  acordo  com  a 
equação (6) ou (7). Logo, L pode ser a espessura de uma barra, o comprimento do lado de um quadrado, ou o diâmetro 
de  um  buraco.  Alguns  materiais,  como  a  madeira,  ou  o  cristal,  dilatam‐se  de  modo  diferente  em  diferentes  direções. 
Não vamos levar em conta esse efeito. 
Convém que você saiba o seguinte a respeito do coeficiente α: 
1) A unidade em que se exprime α é o inverso do grau correspondente à escala considerada. Por exemplo, se estivermos 
trabalhando na escala Celsius, α é expresso na unidade °C−1. 

 

2)  O  coeficiente  de  dilatação  α  é  um  número  da  ordem  de  10−6,  ou  seja,  da  ordem  de  milionésimos.  Por  isso,  nas 
considerações teóricas, abandonamos as potências de α superiores à primeira; com isto estaremos cometendo um erro 
não mensurável experimentalmente. 
Podemos  entender  a  dilatação  térmica  qualitativamente,  em  termos  das  moléculas  do  material.  Imagine  as 
forças interatômicas de um sólido sendo molas, como na figura 9. 

 
FIGURA 9 (a) Podemos visualizar as forças entre os átomos vizinhos em um sólido imaginando‐os interligados por molas, 
com  um  comportamento  análogo  ao  da  mola  que  se  dilata  com  mais  facilidade  do  que  se  comprime.  (b)  Gráfico  da 
energia potencial pela distância entre dois átomos vizinhos, mostrando que as forças não são simétricas. A medida que 
a energia aumenta e os átomos oscilam com maior amplitude, a distancia média aumenta. 
 
Cada  átomo  vibra  em  torno  de  uma  posição  de  equilíbrio.  Quando  a  temperatura  aumenta,  a  energia  e  a 
amplitude das vibrações também aumentam. As forças das molas interatômicas não são simétricas em relação à posição 
de  equilíbrio;  em  geral  elas  se  comportam  como  molas  que  se  dilatam  com  mais  facilidade  do  que  se  comprimem. 
Consequentemente,  quando  a  amplitude  das  vibrações  aumenta,  a  distância  média  entre  as  moléculas  também 
aumenta. À medida que os átomos se afastam, todas as dimensões aumentam. 
A  proporcionalidade  direta  expressa  na  equação  (6)  não  é  exata;  ela  é  aproximadamente  correta  somente 
quando  ocorrem  variações  de  temperatura  muito  pequenas.  Em  um  dado  material,  α  varia  ligeiramente  com  a 
temperatura inicial T0 e com a amplitude do intervalo de temperatura. Vamos, porém, desprezar esse efeito aqui. 
Valores  médios  de  α  para  diversos  materiais  são  listados  na  tabela  1.  Dentro  da  margem  de  precisão  desses 
valores, não precisamos nos preocupar se T0  é 0 °C ou 20 °C, ou alguma outra temperatura. Note que os valores típicos 
de  α  são  muito  pequenos;  mesmo  considerando  uma  variação  de  temperatura  de  100  °C,  a  variação  relativa  do 
comprimento ΔL/L0 é da ordem de apenas 1/1000 para os metais listados na tabela. 
 
 
 
 
 

 
 DILATAÇÃO SUPERFICIAL 
Considere  o  sólido  mostrado  na  figura  10.  Aquecendo‐o,  as  arestas,  as  diagonais  das  faces  e  as  diagonais 
internas dilatam‐se. A dilatação térmica ocorre em todas as direções. Por isso, as superfícies desse corpo, assim como 
seu  volume,  sofrem  dilatações.  Para  muitos  materiais,  o  coeficiente  de  dilatação  linear  é  igual  em  todas  as  direções. 
Nesses  materiais,  a  dilatação  é  chamada  de  isotrópica.  A  dilatação  isotrópica  de  um  sólido  acontece  de  maneira 
semelhante  a  uma  ampliação  fotográfica,  à  exceção  de  que  a  dilatação  é  tridimensional,  enquanto  a  foto  é 
bidimensional. Cada comprimento unitário de reta ou curva do sólido (interna ou superficial) aumenta de um valor igual 
ao coeficiente de dilatação linear para cada grau de aumento de temperatura.  

 

 
 
 
 
 
FIGURA 10 Dilatação de um sólido. 
 

 
Por  isso,  podemos  calcular  a  dilatação  térmica  da  área  de  uma  placa  de  área  inicial  A0  da  seguinte  forma: 
Considere  a  figura  11,  que  mostra  uma  placa  retangular  de  base  a0  e  altura  b0.  A  seguir,  a  placa  é  submetida  a  um 
aumento  de  temperatura  ΔT.  Os  cálculos  das  dilatações  da  base  e  da  altura  estão  indicados  na  figura.  A  dilatação  da 
placa é representada pela área hachurada. 
 
 
 
 
FIGURA 11 Dilatação isotrópica de uma placa. 
 

 
Note que a área final é dada por A = a.b, sendo a = a0(1 + αΔT) e b = b0(1 + αΔT) equação (7).  
Assim A = a.b = a0(1 + αΔT). b0(1 + αΔT) = a0.b0. (1 + αΔT)2 = A0.(1 + 2αΔT + α2ΔT2), como α2  é da ordem de 10‐12 
esse termo pode ser desprezado resultando em 
A = A0.(1 + 2αΔT) = A0 + A02αΔT ou ΔA = A0βΔT    [8] 
Sendo β = 2α. 
O fator β é chamado de coeficiente de dilatação superficial. O seu valor é, com alta precisão, igual ao dobro do 
coeficiente de dilatação linear. Isso pode ser justificado pela isotropia do material da placa. 
DILATAÇÃO VOLUMÉTRICA 
Nos  parágrafos  anteriores,  estudamos  a  dilatação  em  uma  dimensão  (dilatação  linear),  e  a  dilatação  em  duas 
dimensões  (dilatação  superficial).  Vejamos  a  dilatação  em  três dimensões,  ou  seja,  a  dilatação  volumétrica.  Para  isto, 
consideremos um bloco feito de material isótropo e que tenha, a T0, o volume V0. Se a temperatura aumentar para o 
valor genérico T, o bloco passará a ter o volume V como mostra a figura 12.            
 
 
 
 
FIGURA 12 Dilatação isotrópica de um bloco. 
 

     
O volume é definido pelo produto de suas dimensões lineares, a, b e c, tal que V = a.b.c. Então, visto que: 
a = a0(1 + α.ΔT), b = b0(1 + α.ΔT) e c = c0(1 + α.ΔT) temos que: 
V = a0(1 + α.ΔT). b0(1 + α.ΔT). c0(1 + α.ΔT) 
V= a0.b0.c0. (1 + α.ΔT)3  
V = V0[1 + 3. α.ΔT + 3. (α.ΔT)2 + (αΔT)3] 
Como mencionado anteriormente, os  valores dos coeficientes de dilatação α é da ordem  de 10−6 e,  portanto, 
nos levam à conclusão que os dois últimos termos entre colchetes podem ser desprezados, quando comparados com a 
ordem de grandeza do segundo termo. 
Então: 

 

V = V0(1 + 3α.ΔT) ou V = V0(1 + γ.ΔT)       [9] 
Onde usamos o fato de γ = 3α. 
O fator γ se chama coeficiente de dilatação volumétrica. Analogamente ao caso da dilatação linear, γ varia ligeiramente 
com a temperatura, e a equação (9) é uma relação aproximada que só vale para pequenas variações de temperatura. 
Em muitas substâncias, γ diminui em temperaturas baixas. Diversos valores de γ nas vizinhanças da temperatura 
ambiente  são  listados  na  tabela  2.  Note  que  os  valores  para  líquidos  são  geralmente  maiores  do  que  os  valores  para 
sólidos.  

 
Em materiais sólidos, observe que existe uma relação simples entre o coeficiente de dilatação volumétrica γ, o 
coeficiente de dilatação superficial β e o coeficiente de dilatação linear α. Perceba que  
  
              [10] 
1 2 3
DILATAÇÃO DE SÓLIDOS VAZADOS 
Quando um objeto sólido contém um buraco em seu interior, o que ocorre com o tamanho do buraco quando a 
temperatura do objeto aumenta? Um erro muito comum é pensar que quando o objeto se expande o buraco se contrai, 
porque o objeto se expande para dentro do buraco. Na verdade, quando o objeto se dilata, o buraco também se dilata 
(figura  13);  conforme  dissemos  anteriormente;  todas  as  dimensões  lineares  do  objeto  se  dilatam  do  mesmo  modo 
quando a temperatura varia. Caso você ainda não tenha se convencido, pense nos átomos da figura 9a como se fossem 
o contorno de um buraco cúbico. Quando o objeto se expande, os átomos se separam e o buraco aumenta de tamanho.  
 
 
 
 
 
 
FIGURA  13  Quando  um  objeto  passa  por  dilatação 
térmica, quaisquer buracos existentes  no objeto também 
se dilatam. (A dilatação foi exagerada na gravura.) 
 

 
DILATAÇÃO DOS LÍQUIDOS 
Se  tratando  dos  líquidos,  os  estudos  são  realizados  apenas  sobre  a  dilatação  volumétrica,  pelo  fato  de  não 
possuírem forma própria. De fato, a mesma lei que se aplica à dilatação dos sólidos se aplica também a dos líquidos. 
Portanto, as equações matemáticas da dilatação dos sólidos são usadas nos cálculos da dilatação dos líquidos. 
Sendo  V0  o  volume  inicial  de  um  líquido  qualquer,  γ  o  coeficiente  de  dilatação  volumétrica  do  líquido  e  ΔT  a 
variação da temperatura, temos:  
V = V0 + ∆V e ∆V = γ.V0.∆T 
De forma a medir a dilatação volumétrica dos líquidos, utilizamos recipientes sólidos pelo fato de os líquidos não 
possuírem  uma  forma  própria.  Dessa  maneira,  ao  analisarmos  o  comportamento  térmico  dos  líquidos,  devemos 
também considerar a dilatação do recipiente que por sinal ocorre ao mesmo tempo em que a dilatação do líquido. 
Vejamos um  exemplo: imagine um recipiente cheio de um líquido até sua borda como mostra a figura 14. Se 
aquecermos  o  conjunto,  sólido  mais  líquido,  veremos  que  o  líquido  transbordará,  pois os  líquidos  se  dilatam  mais  do 
que os sólidos. A quantidade que transbordou do recipiente nos dá a medida da dilatação aparente do líquido (ΔVap).  
 

 
 
 
FIGURA 14 Após ser aquecido, o líquido transborda 
do recipiente. 
 

 
Se  conhecermos  a  dilatação  do  recipiente  (ΔVrec),  podemos  determinar  a  dilatação  real  do  líquido  (ΔV)  da 
seguinte forma: 
ΔV = ΔVrec+ ΔVap            [11] 
Fazendo uso da equação da dilatação volumétrica, podemos escrever: 
∆Vap = γap.V0.∆T e ∆Vrec = γrec.V0.∆T 
Onde γap é o coeficiente de dilatação aparente do líquido e γrec é o coeficiente de dilatação volumétrica do recipiente. 
Fazendo algumas substituições temos: 
γ = γrec + γap            [12] 
COMPORTAMENTO ANÔMALO DA ÁGUA 
De um modo geral os líquidos se dilatam ao aumentar a temperatura. Há, no entanto, exceções. Entre elas, a 
dilatação  da  água  —  ela  tem  uma  pequena  anomalia  de  consequências  extraordinárias:  seu  volume  diminui  com  a 
diminuição da temperatura até atingir o seu valor mínimo a 4,0 °C; em seguida, mesmo que a temperatura continue a 
diminuir, o volume volta a aumentar até iniciar seu congelamento. A sua densidade tem a variação oposta e atinge seu 
valor máximo a 4,0 °C. Essas variações são mostradas nos dois trechos bem ampliados dos gráficos da figura 15. 

         
FIGURA 15 (a) Gráfico do volume da água em função da temperatura; (b) gráfico da densidade da água em função da 
temperatura. (Ambos sob pressão normal.) 
 
Esse comportamento atípico da água a temperaturas próximas da de solidificação (0 °C), sob pressão normal, 
pode ser explicado pela transição da água líquida, sem estrutura cristalina, a gelo, com uma estrutura cristalina muito 
particular. Veja a figura abaixo: 
 
 
 
FIGURA 16 estrutura cristalina da água a) líquida e b) sólida. 
 

 
Ela  mostra  esquematicamente  a  estrutura  cristalina  do  gelo  sob  pressão  normal.  Como  as  moléculas  da  água 
têm  uma  forma  angular  (veja  o  detalhe),  o  agrupamento  entre  elas  para  formar  o  gelo  resulta  em  uma  estrutura 
cristalina cheia de lacunas. 
Nesse modelo da estrutura molecular do gelo, as esferas maiores vermelhas simbolizam os átomos de oxigênio; 
as  menores,  amarelas,  os  átomos  de  hidrogênio.  Note  que  a  estrutura  cristalina  do  gelo,  embora  tenha  a  rigidez  dos 
sólidos,  é  composta  de  anéis,  por  isso  o  gelo  tem  densidade  menor  do  que  a  água:  a  0  °C,  sob  pressão  normal,  a 
densidade da água é 1 000 kg/m3 e a do gelo é 920 kg/m3. 
 
10 
A  anomalia  ocorre  quando  a  temperatura  da  água  se  aproxima  de  sua  temperatura  de  solidificação.  Isso  se 
explica estatisticamente — uma parte de suas moléculas se “antecipa”, agrupando‐se em cristais de gelo microscópicos 
e menos densos que a água. A presença desses agrupamentos que se multiplicam quando a temperatura da água abaixa 
de 4,0 °C para 0 °C torna a água gradativamente menos densa até que toda a mudança de estado se complete. 
IMPORTÂNCIA DA DIFERENÇA DE DENSIDADE ENTRE A ÁGUA E O GELO 
Em regiões frias, a vida marinha poderia desaparecer caso a água se dilatasse regularmente em toda a faixa de 
temperaturas. Nesses locais, as camadas superiores de um lago se congelam, mas as camadas inferiores continuam no 
estado  líquido,  permitindo  a  manutenção  de  plantas  e  animais  aquáticos.  Nessas  regiões,  quando  o  inverno  inicia,  a 
temperatura ambiente e a temperatura da superfície do lago começam a diminuir. À medida que a água da superfície se 
resfria,  ela  sofre  contração,  torna‐se  mais  densa  e  afunda,  enquanto  a  água  das  camadas  mais  baixas  sobe.  Essas 
correntes  descendentes  e  ascendentes  (chamadas  de  correntes  convectivas)  permitem  que  a  água  do  lago  se  resfrie 
homogeneamente até a temperatura de 4 °C. 
A partir desse ponto, a água da superfície continua se resfriando, pois a temperatura ambiente continua caindo, 
mas a água torna‐se menos densa, pois ela sofre expansão quando a temperatura torna‐se inferior a 4 °C. Por isso, a 
água da superfície flutua sobre as camadas inferiores, que se mantêm a 4 °C (figura 17). O resfriamento da superfície do 
lago continua, e a superfície se congela a 0 °C, atingindo valores muitos graus abaixo de zero.  
 
 
 
 
FIGURA 17 Camada de gelo na superfície do lago sobre o 
mergulhador. 
 

 
 
6 QUANTIDADE DE CALOR  
Quando você coloca uma colher em uma xícara de café quente, a colher esquenta e o café esfria, e eles tendem 
a atingir o equilíbrio térmico. A interação que produz essas variações de temperatura é basicamente uma transferência 
de  energia  entre  uma  substância  e  a  outra.  A  transferência  de  energia  produzida  apenas  por  uma  diferença  de 
temperatura  denomina‐se  transferência  de  calor  ou  fluxo  de  calor,  e  a  energia  transferida  desse  modo  denomina‐se 
calor.  
O estudo da relação entre o calor e outras formas de energia evoluiu gradualmente durante o século XVIII e o 
século XIX. Sir James Joule (1818‐1889) estudou como a água pode ser aquecida ao ser vigorosamente mexida com um 
agitador  (figura  18a).  As  pás  do  agitador  transferem  energia  para  a  água,  realizando  um  trabalho  sobre  ela,  e  Joule 
verificou que o aumento de temperatura é proporcional ao trabalho realizado. A mesma variação de temperatura pode 
também  ser  obtida  colocando‐se  a  água  em  contato  com  algum  corpo  mais  quente  (figura  18b);  logo,  essa  interação 
também  deve  envolver  uma  troca  de  energia.  Em  capítulos  posteriores  discutiremos  a  relação  entre  calor  e  energia 
mecânica mais detalhadamente. 

 
FIGURA 18 A mesma variação de temperatura produzida em um mesmo sistema pode ser obtida (a) realizando‐se um 
trabalho sobre o sistema e (b) transferindo‐se calor para o sistema. 

 
11 
Podemos  definir  uma  unidade  de  quantidade  de  calor  com  base  na  variação  de  temperatura  de  materiais 
específicos. A caloria (abreviada como cal) é definida como a quantidade de calor necessária para elevar a temperatura 
de um grama de água de 14,5 °C até 15,5 °C. A quilocaloria (kcal), igual a 1000 cal, também é uma unidade muito usada; 
a caloria usada para alimentos, também chamada de grande caloria, é na realidade uma quilocaloria.  
Como o calor é uma energia em trânsito, deve existir uma relação entre essas unidades e as unidades de energia 
mecânica que conhecemos, como, por exemplo, o joule. Experiências semelhantes às realizadas por Joule mostraram 
que 
1cal = 4,186J 
1 kcal = 1000 cal = 4186 J 
1 Btu = 778 ft ∙ lb = 252 cal = 1055 J 
A caloria não é uma unidade SI fundamental. O Comitê Internacional de Pesos e Medidas recomenda o uso do 
joule como a unidade básica de todas as formas de energia, inclusive o calor. Seguiremos essa recomendação. 
CALOR ESPECÍFICO 
Usamos o símbolo Q para a quantidade de calor. Quando associada a uma variação infinitesimal de temperatura 
dT, chamamos essa quantidade de dQ. Verifica‐se que a quantidade de calor Q necessária para elevar a temperatura da 
massa  m  de  um  material  de  T1  até  T2  é  aproximadamente  proporcional  à  variação  de  temperatura  ΔT  =  T2  –  T1.  Ela 
também  é  proporcional  à  massa  m  do  material.  Quando  você  aquece  água  para  fazer  o  chá,  precisa  o  dobro  da 
quantidade de calor para fazer duas xícaras em vez de uma, se a variação de temperatura for a mesma. A quantidade de 
calor  também  depende  da  natureza  do  material:  para  elevar  de  1  °C  a  temperatura  de  um  quilograma  de  água  é 
necessário  transferir  uma  quantidade  de  calor  igual  a  4190  J,  enquanto  basta  transferir  910  J  de  calor  para  elevar  a 
temperatura de um quilograma de alumínio de 1 °C. 
Reunindo todas as relações mencionadas, podemos escrever  
Q = mcΔT             [13]  
onde a grandeza c, que possui valores diferentes para cada tipo de material, é denominada calor específico (ou também 
chamada  de  capacidade  calorífica  específica)  do  material.  Para  uma  variação  de  temperatura  infinitesimal  dT  e  uma 
correspondente quantidade de calor dQ, temos, 
dQ = mc dT             [14] 
1 dQ
c             [15] 
m dT
Nas equações (13), (14) e (15), Q (ou dQ) e ΔT (ou dT) podem ter valores positivos ou negativos. Quando esses 
valores são positivos, o calor é transferido para o corpo e sua temperatura aumenta; quando são negativos, o calor é 
libertado pelo corpo e sua temperatura diminui. 
O calor específico da água é aproximadamente igual a 4190 J/kg ∙ K ou 1 cal/g ∙°C ou 1Btu/1b ∙°F 
O  calor  específico  de  um  material  depende  até  certo  ponto  da  temperatura  inicial  e  do  intervalo  de 
temperatura.  A  figura  19  mostra  essa  dependência  no  caso  da  água.  Nos  problemas  e  exemplos  deste  capítulo 
desprezaremos essa pequena variação. 
 
 
 
FIGURA  19  Calor  específico  da  água  em  função  da  temperatura.  O 
valor de c varia menos do que 1% entre 0°C e 100°C. 
 

 
CALOR ESPECÍFICO MOLAR 
Algumas vezes é mais conveniente descrever a quantidade de uma substância em termos do número de moles n 
em vez de  especificar a massa m do material. Lembrando‐se dos seus estudos de química, você sabe que um  mol de 
qualquer substância pura sempre contém o mesmo número de moléculas.  
A letra M indica o mol ou massa molar de qualquer substância. (A grandeza M, algumas vezes, é chamada de 
peso molecular, porém a expressão massa molar é mais apropriada: essa grandeza depende da massa da molécula, e 
não do seu peso.) Por exemplo, a massa molar da água é igual a 18,0 g/mol = 18,0 × 10–3 kg/mol; um mol de água possui 
massa igual a 18,0 g = 0,0180 kg. A massa total m de um material é igual à massa molecular M vezes o número de moles 
n: 

 
12 
m = nM             [16] 
Substituindo a massa m na equação (13) pelo produto nM, achamos 
Q = nMc ΔT             [17]  
O produto Mc denomina‐se calor específico molar (ou simplesmente calor molar) e será designado pela letra C 
(maiúscula). Usando essa notação, podemos reescrever a equação (17) na forma  
Q = nCΔT             [18]  
Comparando  essa  equação  com  a  equação  (15),  podemos  expressar  o  calor específico  molar  C  (calor  por  mol 
por variação da temperatura) em termos de calor específico c (calor por massa por variação da temperatura) e da massa 
molar M (massa por mol): 
1 dQ
C  Mc             [19] 
n dT
Por exemplo, o calor específico molar da água é C = Mc = (0,0180 kg/mol)(4190 J/kg ∙ K) = 75,4 J/mol∙K. 
Na tabela 3 encontramos os valores do calor específico e do calor específico molar de diversas substâncias. Note 
o valor especialmente alto do calor específico da água. 

 
A última coluna da tabela 3 mostra algo interessante. Os calores específicos molares de quase todos os sólidos 
elementares possuem aproximadamente o mesmo valor, cerca de 25 J/mol ∙ K. Essa correlação, denominada regra de 
Dulong  e  Petit  (em  homenagem  aos  seus  descobridores),  é  a  base  de  uma  ideia  importante.  O  número  de  átomos 
contidos em um mol de qualquer elemento é sempre o mesmo. Isso significa que, considerando uma base por átomo, a 
mesma  quantidade  de  calor  é  necessária  para  elevar  em  um  determinado  número  de  graus  a  temperatura  de  todos 
esses  elementos,  embora  as  massas  dos  átomos  sejam  muito  diferentes.  O  calor  necessário  para  produzir  um  dado 
aumento de temperatura depende somente da quantidade de  átomos que a amostra contém, não da massa de cada 
átomo. No próximo capítulo, ao estudarmos do ponto de vista molecular os detalhes do calor específico, veremos por 
que a regra de Dulong e Petit funciona tão bem. 
Medir precisamente o calor específico e o calor específico molar requer uma grande habilidade experimental. 
Normalmente, a quantidade de energia fornecida ao sistema é determinada medindo‐se a corrente elétrica que aquece 
um  fio  enrolado  em  torno  do  material.  A  variação  de  temperatura  ΔT  é  medida  com  um  termômetro  ou  com  um 
termopar  no  interior  do  material.  Isso  parece  simples,  porém  é  necessário  que  se  tome  muito  cuidado  para  evitar 
transferência de calor entre o material e o meio ambiente. As medidas de materiais sólidos normalmente são realizadas 
mantendo‐se  a  pressão  atmosférica  constante;  os  valores  correspondentes  são  denominados  calores  específicos  à 
pressão constante, simbolizados por cP ou CP. Quando a substância é um gás, em geral é mais fácil mantê‐la dentro de 
um recipiente a volume constante; os valores correspondentes denominam‐se calores específicos a volume constante, 
designados por cv ou CV. Para uma dada substância, CV é diferente de CP. Se o sistema pode se expandir à medida que o 
calor é transferido, existe uma troca de energia adicional, porque o sistema realiza um trabalho sobre as vizinhanças. Se 
o  volume  permanece  constante,  o  sistema  não  realiza  nenhum  trabalho.  Nos  gases,  a  diferença  entre  CP  e  CV  é 
significativa. No capítulo de gases estudaremos em detalhe os calores específicos dos gases. 
 
7 CALORIMETRIA E TRANSIÇÕES DE FASES 
Calorimetria significa ‘medida de calor’. Já discutimos a transferência de energia (calor) envolvida em variações 
de  temperatura.  Ocorre  também  transferência  de  calor  nas  transições  de  fase,  tais  como  a  liquefação  do  gelo  ou  a 
 
13 
ebulição da água. Compreendendo essas relações de calor adicionais, podemos analisar vários problemas envolvendo 
quantidade de calor. 
TRANSIÇÕES DE FASE 
Utilizamos  a  palavra  fase  para  designar  qualquer  estado  específico  da  matéria,  tal  como  o  de  um  sólido,  um 
líquido ou um gás. O composto H2O existe na fase sólida como gelo, na fase líquida como água e na fase gasosa como 
vapor d’água. A transição de uma fase para a outra é chamada de transição de fase ou mudança de fase. Em uma dada 
pressão, a transição de fase ocorre em uma temperatura definida, sendo usualmente acompanhada por uma emissão 
ou absorção de calor e por uma variação de volume e de densidade. 
Um exemplo conhecido da transição de fase é a liquefação do gelo. Quando fornecemos calor ao gelo a 0 °C na 
pressão atmosférica normal, a temperatura do gelo não aumenta. O que ocorre é que uma parte do gelo derrete e se 
transforma  em  água  líquida.  Adicionando‐se  calor  lentamente  de  modo  que  seja  mantida  a  temperatura  do  sistema 
muito próxima do equilíbrio térmico, a temperatura do sistema permanece igual a 0 °C até que todo o gelo seja fundido 
(figura 20).  
 
 
 
FIGURA  20  O  ar  circundante  está  à  temperatura  ambiente,  mas 
essa mistura de água e gelo permanece a 0°C até que todo o gelo 
tenha derretido e que a mudança de fase se complete. 
 

 
O calor fornecido a esse sistema não é usado para fazer sua temperatura aumentar, mas sim para produzir uma 
transição de fase de sólido para líquido. São necessários 3,34 x 105 J de calor para converter 1 kg de gelo a 0°C em 1 kg 
de  água  líquida  a  0°C  em  condições  normais  de  pressão  atmosférica.  O  calor  necessário  por  unidade  de  massa 
denomina‐se  calor  de  fusão  (algumas  vezes  chamado  de  calor  latente  de  fusão),  designado  por  Lf.  Para  a  água 
submetida a uma pressão atmosférica normal, o calor de fusão é dado por  
Lf = 3,34 x 105 J/kg = 79,6 cal/g = 143 Btu/lb  
Generalizando,  para  liquefazer  a  massa  m  de  um  sólido  cujo  calor  de  fusão  é  Lf,  é  necessário  fornecer  ao 
material uma quantidade de calor Q dada por 
Q = mLf  
Esse processo é reversível. Para congelar a água líquida a 0°C, devemos retirar calor da água; o módulo do calor 
é  o  mesmo,  mas,  nesse  caso,  Q  é  negativo,  porque  estamos  retirando  calor,  e  não  adicionando.  Para  englobar  essas 
duas possibilidades e para incluir outras transições de fase, podemos escrever 
Q = ±mL             [20]  
O sinal positivo (calor entrando no sistema) é usado quando o sólido se funde; o sinal negativo (calor saindo do 
sistema) é usado quando o líquido se solidifica. O calor de fusão depende do material e também varia ligeiramente com 
a pressão.  
Para  qualquer  material,  a  uma  dada  pressão,  a  temperatura  de  fusão  é  sempre  igual  à  temperatura  de 
liquefação.  Essa  temperatura  única  em  que  a  fase  líquida  coincide  com  a  fase  sólida  (água  e  gelo,  por  exemplo) 
caracteriza uma condição chamada de equilíbrio de fase.  
Podemos  repetir  o  raciocínio  anterior  para  o  caso  da  ebulição  ou  vaporização,  uma  transição  da  fase  líquida 
para a fase gasosa. O calor correspondente (por unidade de massa) denomina‐se calor de vaporização Lv. Sob pressão 
atmosférica normal, o calor de vaporização Lv da água é 
Lv = 2,256 x 106 J/kg = 539 cal/g = 970 Btu/1 b 
Ou  seja,  é  necessário  fornecer  2,256  x  106  J  para  fazer  1  kg  de  água  líquida  se  transformar  em  1  kg  de  vapor 
d'água a 100 °C. Em comparação, o calor necessário para aquecer 1 kg de água de 0°C até 100 °C é dado por Q = mcΔT = 
(1,0 kg)(4190 J/kg∙°C) x (100 °C) = 4,19 x105 J, menos do que um quinto do calor necessário para a vaporização da água a 
100 °C. Esse resultado está de acordo com nossa experiência cotidiana na cozinha: uma panela com água pode atingir a 
temperatura  de ebulição em alguns minutos, porém é necessário um tempo muito maior para fazer a água vaporizar 
completamente.  
Como  a  fusão,  a  ebulição  é  uma  transição  de  fase  reversível.  Quando  retiramos  calor  de  um  vapor,  na 
temperatura de ebulição, o gás retoma para a fase líquida, ou se condensa, cedendo ao ambiente a mesma quantidade 
de  calor  que  foi  necessária  para  vaporizá‐lo  (calor  de  vaporização).  A  uma  dada  pressão,  a  temperatura  de  ebulição 

 
14 
coincide  com  a  temperatura  de  condensação;  nessa  temperatura  existe  um  equilíbrio  de  fase  no  qual  a  fase  líquida 
coexiste com a fase gasosa. 
Tanto Lv quanto a temperatura de ebulição de um dado material dependem da pressão. Em um local elevado, 
como Itatiaia, por exemplo, onde a pressão atmosférica média é menor do que a do nível do mar, a água ferve a uma 
temperatura menor (cerca de 95 °C) do que a temperatura de ebulição da água em um local no nível do mar (100 °C). 
Nessa pressão menor, o calor de vaporização, aproximadamente igual a 2,27 x 106 J/kg, é ligeiramente maior do que o 
calor de vaporização à pressão atmosférica normal.  
Na tabela 4 fornecemos o calor de fusão e de vaporização de diversas substâncias e as respectivas temperaturas 
de  fusão  e  ebulição  sob  pressão  atmosférica  normal.  Pouquíssimos  elementos  possuem  temperaturas  de  fusão  nas 
vizinhanças da temperatura ambiente; para citar um elemento, citamos o gálio metálico que tem sua temperatura de 
fusão em torno de 29,76 °C. 

 
A figura 21 mostra como a temperatura varia quando fornecemos calor continuamente a uma amostra de gelo 
com temperatura inicial abaixo de 0 °C (ponto a). A temperatura aumenta até ser atingido o ponto de fusão (ponto b). A 
seguir, enquanto se fornece mais calor, a temperatura permanece constante até que todo gelo seja fundido (ponto c). 
Então  a  temperatura  volta  a  subir  novamente  até  atingir  a  temperatura  de  ebulição  (ponto  d).  Nesse  ponto,  a 
temperatura permanece constante até que toda a água seja transformada em vapor d’água (ponto e). Caso a taxa de 
fornecimento de calor seja constante, a reta referente ao aquecimento da fase sólida (gelo) é mais inclinada do que a 
reta referente à fase líquida (água). Você sabe por quê? (Veja a tabela 3.) 
Em  certas  circunstâncias,  uma  substância  pode  passar  diretamente  da  fase  sólida  para  a  fase  gasosa.  Esse 
processo denomina‐se sublimação, e dizemos que o sólido sublima. O calor de transição correspondente denomina‐se 
calor de sublimação, LS. O dióxido de carbono líquido não pode existir a uma pressão menor do que cerca de 5 × 105  Pa 
(cerca de 5 atm), e o ‘gelo seco’ (dióxido de carbono sólido) sublima na pressão atmosférica. A sublimação da água em 
um alimento congelado produz fumaça em uma geladeira. O processo inverso, uma transição da fase vapor para a fase 
sólida,  ocorre  quando  se  forma  gelo  sobre  a  superfície  de  um  corpo  frio,  tal  como  no  caso  da  serpentina  de  um 
refrigerador. 
 
 
 
 
FIGURA  21  Gráfico  da  temperatura  em  função  do  tempo 
de  uma  amostra  de  água  inicialmente  na  fase  sólida 
(gelo).  O  calor  é  fornecido  à  amostra  a  taxa  constante.  A 
temperatura  permanece  constante  durante  todas  as 
mudanças  de  fase,  desde  que  a  pressão  permaneça 
constante. 
 

 
 
15 
A água muito pura pode ser resfriada até diversos graus abaixo do ponto de congelamento sem se solidificar; o 
estado de equilíbrio instável resultante denomina‐se super‐resfriado. Quando jogamos nessa água um pequeno cristal 
de gelo, ou quando a agitamos, ela se cristaliza em um segundo ou em uma fração de segundo. O vapor d’água super‐
resfriado condensa rapidamente, formando gotículas de névoa na presença de alguma perturbação, como partículas de 
poeira ou radiações ionizantes. Esse princípio é usado na chamada ‘semeadura de nuvens’ (bombardeio de nuvens com 
nitrato de prata), que geralmente possuem vapor d’água super‐resfriado, provocando a condensação e a chuva. 
Algumas vezes um líquido pode ser superaquecido acima da sua temperatura de ebulição normal. Novamente, 
qualquer perturbação pequena, tal como a agitação do líquido ou a passagem de partículas carregadas em seu interior, 
produz ebulição local com a formação de bolhas. 
Os  sistemas  de  aquecimento  a  vapor  de  edifícios  utilizam  processos  de  vaporização  e  condensação  para 
transferir  calor  do  aquecedor  para  os  radiadores.  Cada  quilograma  de  água  que  se  transforma  em  vapor  no  boiler 
(aquecedor) absorve cerca de 2 × 10–6 J (o calor de vaporização Lv da água) do boiler e liberta essa mesma quantidade 
quando  se  condensa  nos  radiadores.  Os  processos  de  vaporização  e  de  condensação  também  são  usados  em 
refrigeradores, condicionadores de ar e em bombas de calor.  
Os  mecanismos  de  controle  de  temperatura  de  muitos  animais  de  sangue  quente  são  baseados  no  calor  de 
vaporização, removendo o calor do corpo ao usá‐lo na vaporização da água da língua (respiração arquejante) ou da pele 
(transpiração).  O esfriamento produzido pela vaporização possibilita a manutenção da temperatura constante do corpo 
humano  em  um  deserto  seco  e  quente,  onde  a  temperatura  pode  atingir  até  55  °C.  A  temperatura  da  pele  pode 
permanecer  até  cerca  de  30  °C  mais  fria  do  que  a  temperatura  do  ar  ambiente.  Nessas  circunstâncias,  uma  pessoa 
normal  perde  vários  litros  de  água  por  dia  por  meio  da  transpiração,  e  essa  água  precisa  ser  reposta.  Veteranos  que 
fazem excursões em desertos, afirmam que, no deserto, qualquer cantil de menos de um galão (aproximadamente 3,78 
litros) seria visto como um brinquedo! O resfriamento produzido pela vaporização explica também por que você sente 
frio ao sair de uma piscina. 
O resfriamento produzido pela vaporização é também usado para resfriar edifícios em climas secos e quentes, e 
para  fazer  condensar  e  reciclar  o  vapor  ‘usado’  em  usinas  geradoras  termelétricas  com  aquecimento  produzido  pela 
queima de carvão ou por reações termonucleares. É isso que ocorre naquelas enormes torres de concreto que você vê 
nas vizinhanças dessas usinas.  
Reações  químicas,  tais  como  uma  combustão,  são  análogas  a  uma  transição  de  fase  no  que  diz  respeito  ao 
envolvimento de uma troca de calor. A combustão completa de um grama de gasolina produz cerca de 46000 J, ou cerca 
de 11000 cal, de modo que o calor de combustão Lc de gasolina é dado por 
Lc = 46000 J/g = 4,6 × 107 J/kg  
Os valores associados à energia dos alimentos podem ser definidos de modo semelhante; a unidade de energia 
obtida de um alimento, embora seja chamada de caloria, é na verdade a quilocaloria, igual a 1000 cal ou 4186 J. Quando 
dizemos  que  um  grama  de  manteiga  “contém  6  calorias”,  queremos  dizer  que  ela  libera  6  kcal  de  calor  (6000  cal  ou 
25000  J)  quando  ocorre  uma  reação  entre  o  oxigênio  e  os  átomos  de  carbono  e  de  hidrogênio  da  manteiga  (com  o 
auxílio  de  enzimas),  e  os  produtos  da  reação  são  o  CO2  e  H2O.  Nem  toda  essa  energia  é  utilizada  diretamente  para 
produzir trabalho mecânico útil. 
PRINCÍPIO GERAL DAS TROCAS DE CALOR 
Considere um processo calorimétrico em que dois corpos A e B trocam entre si 70 calorias, desde o instante em 
que foram colocados em presença um do outro até ser atingido o equilíbrio térmico. Admitindo não haver perdas para o 
ambiente  e  tendo  em  vista  o  princípio  da  conservação  da  energia,  se  o  corpo  A  perdeu  calor  e  o  corpo  B  recebeu, 
podemos escrever, de acordo com a convenção já estabelecida: 
QA = ‐70 cal e QB = 70 cal 
Portanto, o corpo A perdeu 70 calorias e o corpo B recebeu 70 calorias. Perceba que as quantidades de calor (QA 
perdida pelo corpo A e QB recebida pelo corpo B) são iguais em módulo, mas possuem sinais contrários. Então: 
QA = ‐QB ou QA + QB = 0 
Esta  conclusão  é  válida  para  qualquer  número  de  corpos  que  troquem  calor  até  se  estabelecer  o  equilíbrio 
térmico. Podemos então enunciar o seguinte princípio geral das trocas de calor:  
Quando dois ou mais corpos trocam calor entre si, até atingirem o equilíbrio térmico, a soma algébrica das quantidades 
de calor trocadas é nula. 
A generalização para qualquer número de corpos permite escrever: 
Q1 + Q2 + Q3 + Q4 + … = 0 ou  ∑Q = 0 
As  trocas  de  calor  nos  processos  calorimétricos  desenvolvidos  em  laboratório  geralmente  ocorrem  com  os 
corpos colocados no interior de recipientes denominados calorímetros (figura 22). 

 
16 
 
 
 
 
 
 
FIGURA 22 Esquema, em corte, de um calorímetro. 
 

 
Um  bom  calorímetro  deve  ser,  dentro  do  possível,  isolado  termicamente  do  ambiente,  para  que  se  possa 
desprezar a  parcela de calor que inevitavelmente se perde. Além disso, deve  ter uma capacidade térmica baixa,  para 
não interferir no equilíbrio térmico que se estabelece. Se tal não acontecer e o calorímetro trocar uma quantidade de 
calor  apreciável,  sua  capacidade  térmica  C  deve  ser  levada  em  conta  nos  cálculos.  Nesse  caso,  a  quantidade  de  calor 
trocada pelo calorímetro será dada por: 
Qcalorím = C ΔT 
A  capacidade  térmica  do  calorímetro  também  costuma  ser  chamada  de  equivalente  em  água  do  calorímetro, 
representado  pela  massa  de  água  que  sofreria  a  mesma  variação  de  temperatura  do  calorímetro.  Assim,  dizer  que  o 
equivalente em água do calorímetro é 5 gramas significa dizer que sua capacidade térmica é 5 cal/°C. Realmente, para a 
quantidade de água considerada (m = 5 g), sendo o calor específico c = 1 cal/(g.°C), a capacidade térmica vale: 
C = m c = 5 . 1  ⇒ C = 5 cal/°C 
 
8 MECANISMOS DE TRANSFERÊNCIA DE CALOR 
Já falamos sobre condutores, materiais que permitem a condução de calor, e isolantes, materiais que impedem 
a transferência de calor entre corpos. Vamos agora examinar com mais detalhes as taxas de transferência de energia. 
Na cozinha, você usa uma panela de alumínio para uma boa transferência de calor entre o fogão e o interior da panela, 
enquanto  a  parede  da  geladeira  é  feita  com  um  material  que  impede  a  transferência  de  calor  para  o  interior  da 
geladeira. Como você pode descrever a diferença entre esses dois materiais? 
Os três mecanismos de transferência de calor são a condução, a convecção e a radiação. A condução ocorre no 
interior de um corpo ou entre dois corpos em contato. A convecção depende do movimento da massa de uma região 
para outra. A radiação é a transferência de calor que ocorre pela radiação eletromagnética, tal como a luz solar, sem 
que seja necessária a presença de matéria no espaço entre os corpos. 
CONDUÇÃO 
Quando  você  segura  uma  das  extremidades  de  uma  barra  de  cobre  e  coloca  a  outra  extremidade  sobre  uma 
chama,  a  extremidade  que  você  está  segurando  fica  cada  vez  mais  quente,  embora  ela  não  esteja  em  contato  direto 
com  a  chama.  O  calor  é  transferido  por  condução  através  do  material  até  atingir  a  extremidade  mais  fria.  Em  nível 
atômico,  verificamos  que  os  átomos  de  uma  região  quente  possuem  em  média  uma  energia  cinética  maior  do  que  a 
energia cinética dos átomos de uma região vizinha. As colisões desses átomos com os átomos vizinhos fazem com que 
eles lhes transmitam parte da energia. Os átomos vizinhos colidem com outros átomos vizinhos, e assim por diante, ao 
longo  do  material.  Os  átomos,  em  si,  não  se  deslocam  de  uma  região  a  outra  do  material,  mas  a  energia  cinética  se 
desloca. 
Quase  todos  os  metais  utilizam  outro  mecanismo  mais  eficiente  para  conduzir  o  calor.  No  interior  do  metal, 
alguns elétrons se libertam dos seus átomos originais e ficam vagando pela rede cristalina. Esses elétrons ‘livres’ podem 
transferir  energia  rapidamente  da  região  mais  quente  para  a  região  mais  fria  do  metal.  É  por  isso  que  os  metais 
geralmente  são  bons  condutores  de  calor.  Uma  barra  de  metal  a  20  °C  parece  estar  mais  fria  do  que  um  pedaço  de 
madeira  a  20  °C  porque  o  calor  pode  fluir  mais  facilmente  entre  sua  mão  e  o  metal.  A  presença  de  elétrons  ‘livres’ 
também faz com que os metais sejam bons condutores de eletricidade. 
A  transferência  de  calor  ocorre  somente  entre  regiões  com  temperaturas  diferentes,  e  o  sentido  de 
transferência de calor é sempre da temperatura maior para a temperatura menor. A figura 23a mostra uma barra de um 
material condutor de comprimento L com uma seção reta de área A. A extremidade esquerda da barra é mantida a uma 
temperatura TH, e a extremidade direita é mantida a uma temperatura mais baixa TC; isso faz com que o calor flua da 

 
17 
esquerda para a direita. Os lados da barra são cobertos por um isolante ideal, de modo que o calor não pode fluir por 
eles. 

 
FIGURA 23 Fluxo de calor constante produzido pela condução de calor em uma barra uniforme. 
 
Quando uma quantidade de calor dQ é transferida através da barra em um tempo dt, a taxa de transferência de 
calor  é  dada  por  dQ/dt.  Chamamos  essa  grandeza  de  taxa  de  transferência  de  calor  ou  corrente  de  calor,  e  a 
designamos por H. Ou seja, H = dQ/dt. A experiência mostra que a taxa de transferência de calor é proporcional à área A 
da seção reta da barra (figura 23b) e a diferença de temperatura (TH – TC), e inversamente proporcional ao comprimento 
da  barra  L  (figura  23c).  Introduzindo  uma  constante  de  proporcionalidade  k,  denominada  condutividade  térmica  do 
material, temos 
dQ T T
H  kA H C           [21] 
dt L
A quantidade (TH – TC)/L é a diferença de temperatura por unidade de comprimento, e é denominada módulo do 
gradiente de temperatura. O valor numérico de k depende do material da barra. Os materiais com valores elevados de k 
são  bons  condutores  de  calor;  os  materiais  com  valores  pequenos  de  k  conduzem  pouco  calor  ou  são  isolantes.  A 
equação (21) também fornece a taxa de transferência de calor através de uma placa, ou de qualquer corpo homogêneo 
que possua uma seção reta A ortogonal à direção do fluxo de calor; L é o comprimento da trajetória do fluxo do calor. 
As unidades de taxa de transferência de calor são as unidades de energia por tempo, ou potência; a unidade SI 
para a taxa de transferência de calor é o watt (1 W = 1 J/s). Podemos achar as unidades de k explicitando k0 na equação 
(21). Convidamos você a verificar que as unidades SI de k são W/m∙ K. Alguns valores de k são apresentados na tabela 5. 
A condutividade térmica do ar 'morto' (ou seja, ar em repouso) é muito pequena. Um agasalho de lã mantém 
você quente porque aprisiona o ar entre suas fibras. De fato, muitos materiais isolantes, tais como o isopor ou a fibra de 
vidro,  contêm  grande  quantidade  de  ar  morto.  A  figura  24  mostra  um  material  cerâmico  com  propriedades  térmicas 
bastante incomuns, inclusive uma condutividade térmica muito pequena. 
 
FIGURA  24  Esta  placa  protetora,  desenvolvida  para  ser  usada  no  ônibus 
espacial,  possui  propriedades  térmicas  extraordinárias.  A  condutividade 
térmica extremamente pequena e o calor especifico pequeno desse material 
permitem  que  se  segure  a  placa  pela  parte  lateral,  embora  a  elevada 
temperatura  do  seu  centro  seja  suficiente  para  produzir  a  luz  usada  nesta 
fotografia. 
 
 

 
 
18 
Se a temperatura varia de modo não uniforme ao longo do comprimento da barra não condutora, introduzimos 
uma  coordenada  x  ao  longo  do  comprimento  e  escrevemos  o  gradiente  de  temperatura  na  forma  geral  dT/dx.  A 
correspondente generalização da equação (21) é dada por 
dQ dT
H  kA           [22] 
dt dx
O sinal negativo mostra que o fluxo de calor ocorre sempre no sentido da diminuição da temperatura. 
No isolamento térmico de edifícios, os engenheiros usam o conceito de resistência térmica, designado por R. A 
resistência térmica R de uma placa de material com área A é definida de modo que a taxa de transferência de calor H 
seja dada por 
A(TH  TC )
H             [23] 
R
onde TH e TC são as temperaturas das duas faces da placa. Comparando essa relação com a equação (21), vemos que R é 
dado por 
L
R               [24] 
k
onde L é a espessura da placa. A unidade SI de R é 1 m2 ∙ K/W. Dobrando‐se a espessura da placa, o valor de R também 
dobra. Uma prática comum nas novas construções em climas muito frios do hemisfério norte é empregar valores de R 
em torno de 30 para paredes externas e tetos. Quando o material isolante é disposto em camadas, como no caso de 
paredes compostas, isolamento com fibra de vidro e parte externa com madeira, os valores de R são somados.  
CONVECÇÃO 
A convecção é a transferência de calor ocorrida pelo movimento da massa de uma região do fluido para outra 
região. Exemplos familiares incluem os sistemas de aquecimento de água em residências, o sistema de refrigeração do 
motor de um automóvel e o fluxo do sangue pelo corpo. Quando o fluido é impulsionado pela ação de um ventilador ou 
de uma bomba, o processo denomina‐se convecção forçada; quando o escoamento é produzido pela existência de uma 
diferença de densidade provocada por uma expansão térmica, tal como a ascensão do ar quente, o processo denomina‐
se convecção natural ou convecção livre (figura 25). 
 
 
 
FIGURA 25 Convecção natural ou convecção livre. 

 
A convecção natural na atmosfera desempenha um papel dominante na determinação das condições climáticas 
ao longo do dia, e a convecção nos oceanos é um importante mecanismo de transferência de calor no globo terrestre. 
Em  uma  escala  menor,  os  pilotos  de  planadores  e  as  águias  utilizam  as  correntes  de  ar  ascendentes  oriundas  do 
aquecimento  da  Terra.  O  mecanismo  mais  importante  para  a  transferência  de  calor  no  corpo  humano  (utilizado  para 
manter  a  temperatura  do  corpo  constante  em  diferentes  ambientes)  é  a  convecção  forçada  do  sangue,  na  qual  o 
coração desempenha o papel de uma bomba. 
A transferência de calor por convecção é um processo muito complexo, e não existe nenhuma equação simples 
para descrevê‐lo. A seguir assinalamos alguns fatos experimentais. 
1. A taxa de transferência de calor por convecção é diretamente proporcional à área da superfície. É por essa razão que 
se usa uma área superficial grande em radiadores e aletas de refrigeração. 
2.  A  viscosidade  do  fluido  retarda  o  movimento  da  convecção  natural  nas  vizinhanças  de  superfícies  estacionárias, 
dando origem a uma película ao longo da superfície que, quando é vertical, costuma ter aproximadamente o mesmo 
valor isolante que 1,3 cm de madeira compensada (R = 0,7). A convecção forçada provoca uma diminuição da espessura 
dessa película, fazendo aumentar a taxa de transferência de calor. Isso explica por que você sente mais frio quando há 
um vento frio do que quando o ar está em repouso à mesma temperatura. 
3. Verifica‐se que a taxa de transferência de calor na convecção é aproximadamente proporcional à potência de 5/4 da 
diferença de temperatura entre a superfície e um ponto do seio do fluido. 
RADIAÇÃO 
Radiação  é  a  transferência  de  calor  por  meio  de  ondas  eletromagnéticas,  como  a  luz  visível,  a  radiação 
infravermelha e a radiação ultravioleta. Todo mundo já sentiu o calor da radiação solar e o intenso calor proveniente de 
uma churrasqueira ou das brasas do carvão de uma lareira. A maior parte do calor proveniente desses corpos quentes 
 
19 
atinge você por radiação, e não por condução ou convecção do ar. O calor seria transferido a você mesmo que existisse 
apenas vácuo entre você e a fonte de calor. 
Qualquer corpo, mesmo a uma temperatura normal, emite energia sob a forma de radiação eletromagnética. A 
uma  temperatura  normal,  digamos  a  20  °C,  quase  toda  energia  é  transportada  por  ondas  infravermelhas  com 
comprimentos de onda maior do que os da luz visível (figura 26).  
 
 
FIGURA 26 Esta imagem em infravermelho com cores falsas revela a 
radiação emitida por várias partes do corpo humano. A emissão mais 
forte  (partes  escuras  do  rosto)  vem  das  áreas  mais  quentes, 
enquanto,  por  outro  lado,  há  muito  poucas  emissões  vindas  da 
garrafa de bebida gelada. 

 
À medida que a temperatura aumenta, os comprimentos de onda passam a ter valores menores. A 800 °C, um 
corpo emite radiação visível em quantidade suficiente para adquirir luminosidade própria e parecer vermelho, embora 
mesmo nessa temperatura a maior parte da energia seja transportada por ondas infravermelhas. A uma temperatura de 
3000  °C,  a  temperatura  característica  do  filamento  de  uma  lâmpada  incandescente,  a  radiação  contém  luz  visível 
suficiente para que o corpo pareça 'branco‐quente'. 
A taxa de radiação de energia de uma superfície é proporcional à área A. A taxa aumenta muito rapidamente 
com  a  temperatura,  já  que  é  proporcional  à  quarta  potência  da  temperatura  absoluta  (Kelvin).  Essa  taxa  também 
depende da natureza da superfície. Essa dependência é descrita por uma grandeza “e”, denominada emissividade, que é 
um  número  sem  dimensões  compreendido  entre  0  e  1,  representando  a  razão  entre  a  taxa  de  radiação  de  uma 
superfície  particular  e  a  taxa  de  radiação  de  uma  superfície  de  um  corpo  ideal  com  a  mesma  área  e  a  mesma 
temperatura.  A  emissividade  também  depende  ligeiramente  da  temperatura.  Logo,  a  taxa  de  radiação  H  =  dQ/dt  de 
uma superfície de área A, com uma temperatura T e emissividade e, pode ser expressa pela relação 
H = AeσT4             [25] 
onde σ é uma constante física fundamental denominada constante de Stefan‐Boltzmann. Essa relação denomina‐se lei 
de Stefan‐Boltzmann em homenagem aos seus descobridores, que viveram no final do século XIX. O valor numérico de σ 
com melhor precisão atualmente conhecido é 
σ = 5,670400 × 10–8 W/m2∙K4 
A  emissividade  (e)  de  uma  superfície  escura  é  geralmente  maior  do  que  a  de  uma  superfície  clara.  A 
emissividade de uma superfície lisa de cobre é igual a aproximadamente 0,3, porém o valor de “e” para uma superfície 
negra pode aproximar‐se de um. 
RADIAÇÃO E ABSORÇÃO 
Enquanto  um  corpo  com  temperatura  T  está  irradiando,  o  ambiente  que  está  a  uma  temperatura  Ts  também 
está  irradiando,  e  o  corpo  absorve  uma  parte  dessa  radiação.  Caso  ele  esteja  em  equilíbrio  térmico  com  o  meio 
ambiente, T = Ts, e a taxa de emissão da radiação é igual à taxa de absorção. Para isso ser verdade, a taxa de absorção 
deve ser dada em geral por H = AeσTs4. Então, a taxa de radiação resultante de um corpo a uma temperatura T imerso 
em um ambiente que está a uma temperatura Ts é dada por 
Htotal = AeσT 4 − AeσTs4 = Aeσ(T4 – Ts4)       [26] 
Nessa equação, um valor positivo de H significa que o fluxo resultante ocorre para fora do corpo. A equação (26) 
mostra  que,  na  radiação,  assim  como  na  convecção  e  na  condução,  a  taxa  de  transferência  de  valor  depende  da 
diferença entre as temperaturas de dois corpos. 
APLICAÇÕES DA RADIAÇÃO 
A  transferência  de  calor  por  radiação  é  importante  em  alguns  cenários  surpreendentes.  Um  bebê  em  uma 
incubadora pode esfriar perigosamente se as paredes da incubadora estiverem frias, mesmo que o ar em seu interior 
esteja quente. Algumas incubadoras regulam a temperatura do ar medindo a temperatura da pele do bebê. 
Um corpo que absorve bem o calor também emite bem o calor. Um irradiador ideal, com emissividade igual a 
um,  também  é  absorvedor  ideal,  absorvendo  toda  radiação  que  incide  sobre  ele.  Tal  superfície  ideal  chama‐se  corpo 
negro ideal, ou simplesmente corpo negro. Reciprocamente, um refletor ideal, que não absorve nenhuma radiação, é 
também um irradiador ineficiente. 
Essa é a razão do uso de uma película de prata no interior de uma garrafa com vácuo entre as paredes externas 
(garrafa ‘térmica’), inventada por Sir James Dewar (1842‐1923). Uma garrafa térmica possui uma parede dupla de vidro. 

 
20 
O ar é bombeado para fora do espaço entre essas paredes; isso elimina quase todo o calor transmitido por condução e 
por convecção. A película de prata nas paredes internas provoca a reflexão da maior parte da radiação proveniente do 
interior da garrafa, fazendo a radiação voltar para seu interior, e a própria parede é um emissor muito pobre. Portanto, 
uma garrafa térmica pode manter o café ou a sopa aquecida durante horas. O recipiente conhecido como dewar, usado 
para armazenar gases liquefeitos frios, funciona com base nesse mesmo princípio. 
 
EXERCÍCIOS RESOLVIDOS 
 
01  Quando  se  mede  a  temperatura  do  corpo  humano  com  um  termômetro  clínico  de  mercúrio  em  vidro,  procura‐se 
colocar  o  bulbo  do  termômetro  em  contato  direto  com  regiões  mais  próximas  do  interior  do  corpo  e  manter  o 
termômetro assim durante algum tempo, antes de fazer a leitura. Porque esses dois procedimentos são necessários? 
SOLUÇÃO  
Por  meio  da  transpiração,  a  pele  regula  a  temperatura  interna  do  corpo  humano.  Assim,  para  obter  o  valor  dessa 
temperatura,  devemos  introduzir  o  termômetro  em  uma  das  aberturas  do  corpo,  como,  por  exemplo,  a  boca.  O 
termômetro deve ficar algum tempo em contato com o corpo para que a transferência de calor possa proporcionar o 
equilíbrio térmico entre o mercúrio (do termômetro) e o interior desse corpo humano. 
 
02 Você introduz um termômetro em uma panela de água quente e registra a leitura. Que temperatura você registrou?  
a) a temperatura da água;  
b) a temperatura do termômetro;  
c) uma média aritmética das temperaturas da água e do termômetro;  
d)  uma  média  ponderada  das  temperaturas  da  água  e  do  termômetro,  sendo  que  o  peso  da  temperatura  da  água  é 
maior;  
e)  uma  média  ponderada  das  temperaturas  da  água  e  do  termômetro,  sendo  que  o  peso  da  temperatura  do 
termômetro é maior. 
SOLUÇÃO 
b)  Um  termômetro  composto  por  um  líquido  em  um  tubo  na  verdade  mede  a  sua  própria  temperatura.  Se  o 
termômetro permanecer na água quente por um tempo longo o bastante, ele entrará em equilíbrio térmico com a água, 
e sua temperatura será a mesma da água. 
 
03 Consideremos, por exemplo, que no verão a temperatura máxima atingida na cidade de Fortaleza, no Ceará, foi de 
40 °C. Qual a correspondente temperatura na escala Fahrenheit? 
SOLUÇÃO 
Temos então que TC = 40 °C. Usando a fórmula de conversão, teremos: 
5 (TF ‐ 32) = 9 TC    5 TF ‐ 160 = 9.40    5 TF = 360 + 160    5 TF = 520    TF = 104 °F 
Repare  que,  como  não  estamos  acostumados  com  a  escala  Fahrenheit,  soa  estranho  falarmos  num  valor  de 
temperatura maior que 100° para a temperatura ambiente. Mas, na verdade, isso está absolutamente correto. 
 
04 Um termômetro é graduado em uma escala X, adotando‐se os valores ‐ 10 para o ponto do gelo (fusão do gelo) e 110 
para o ponto do vapor (ebulição da água sob pressão normal). Estabeleça uma fórmula de conversão entre as indicações 
desse termômetro e as indicações de outro termômetro graduado na escala Celsius. 
SOLUÇÃO 
Comparando as escalas dos dois termômetros, teremos: 
 
 
 
 
Tx  (10) T 0 T  10) TC
 C  x   Tx  1,2TC  10  
110  (10) 100  0 120 100

 
 

 
21 
05 Você coloca um pedaço de gelo na boca. O gelo, à temperatura T1 = 32°F, acaba sendo todo convertido em água à 
temperatura do corpo T2 = 98,6°F. Expresse essas temperaturas em °C e K e calcule ΔT = T2 ‐ T1 nas duas escalas. 
SOLUÇÃO 
Inicialmente achamos as temperaturas na escala Celsius. Sabemos que  
 32  32 TC
TF  32 TC para TF  32F : 9

5
 TC  0C
 então   
9 5 para T  98,60F : 98,60  32  TC  T  37C
 F
9 5
C

Para obter a temperatura na escala Kelvin, basta somar 273,15 K ao valor da temperatura em graus Celsius: T1  = 273,15 
K  e  T2  =  310,15  K.  A  temperatura  ‘normal’  do  corpo  humano  é  da  ordem  de  37  °C,  porém,  se  o  médico  disser  que  a 
temperatura do seu corpo é 310 K, não fique preocupado. A diferença de temperatura é dada por ΔT = T2 − T1 = 37 °C = 
37 K. 
 
06  Um  agrimensor  usa  uma  fita  de  aço  de  50000  m  de  comprimento  a  uma  temperatura  de  20  °C.  Qual  é  o 
comprimento da fita em um dia de verão quando a temperatura é igual a 35 °C? 
SOLUÇÃO 
A variação de temperatura é ΔT = T − T0 = 15 °C. 
Então, pela relação ΔL = L0αΔT, a variação de comprimento ΔL e o comprimento final L = L0 + ΔL são 
ΔL = L0α ΔT = (50 m)(1,2.10–5K–1) (15 K) = 9,0.10–3 m = 9,0 mm 
L = L0 + ΔL = 50000 m + 0,009 m = 50009 m. 
Portanto, o comprimento a 35 °C é igual a 50009 m. 
 
07  O  diagrama  abaixo  mostra  a  variação  ΔL  sofrida  por  uma  barra  metálica  de  comprimento  inicial  igual  a  10  m  em 
função da variação de temperatura ΔT. Qual o valor do coeficiente de dilatação linear do material dessa barra? 

 
SOLUÇÃO 
Como: ΔL = L0 α ΔT, temos 
16 = 10 000 ∙ α ∙ 100 
α = 1,6 ∙ 10–5 °C–1 
 
08  À  temperatura  de  15  °C,  encontramos  uma  chapa  de  cobre  com  superfície  de  área  100,0  cm2.  Que  área  terá  essa 
superfície se a chapa for aquecida até 515 °C? 
Dado: coeficiente de dilatação superficial do cobre = 3,2 ∙ 10–5 °C–1 
SOLUÇÃO 
ΔA = A0 β Δθ 
ΔA = 100,0 ∙ 3,2 ∙ 10–5 ∙ (515 – 15) 
ΔA = 1,6 cm2 
Portanto: 
A = A0 + ΔA  ⇒ A = 100,0 + 1,6 (cm2)  ⇒ A = 101,6 cm2 
 
09 Um frasco de vidro com volume igual a 200 cm3 a 20 °C está cheio de mercúrio até a borda. Qual é a quantidade de 
mercúrio  que  transborda quando  a  temperatura  do  sistema  se eleva  até  100  °C?  O  coeficiente  de  dilatação  linear  do 
vidro é igual a 0,40.10–5 K–1. 
SOLUÇÃO 
O coeficiente de dilatação volumétrica do vidro é 
βvidro = 3αvidro = 3 (0,40.10–5 K–1) = 1,2.10–5 K–1 
O aumento de volume do frasco de vidro é 

 
22 
ΔVvidro = βvidro V0 ΔT = (1,2.10–5K–1)(200 cm3)(100 °C – 20 °C) = 0,19 cm3 
O aumento de volume do mercúrio é 
ΔVmercúrio = βmercúrio V0 ΔT = (18.10–5K–1)(200 cm3)(100 °C – 20 °C) = 2,9 cm3 
O volume do mercúrio que transborda é 
ΔVmercúrio − ΔVvidro = 2,9 cm3 − 0,19 cm3 = 2,7 cm3 
 
10  Um  recipiente  de  vidro  de  200  cm3  contém  190  cm3  de  mercúrio  a  0  °C.  Qual  a  máxima  temperatura  a  que  esse 
conjunto pode ser levado sem que haja transbordamento do mercúrio? (Dados: γHg = 1,8 . 10‐4 °C‐1; γvidro = 1,2 . 10‐5 °C‐1, 
coeficientes de dilatação volumétrica do mercúrio e do vidro.) 
SOLUÇÃO 
A  temperatura  limite  para  que  não  haja  transbordamento  é  aquela  em  que  o  volume  do  mercúrio  (VHg)  se  iguala  ao 
volume do recipiente de vidro (Vvidro). Logo  
VHg = Vvidro. 
Como V = V0 + ΔV, podemos escrever: VHg = Vvidro ⇒ V0Hg + ΔVHg = V0vidro + ΔVvidro 
ΔVHg ‐ ΔVvidro = V0vidro ‐ V0Hg  * 
Substituindo a expressão ΔV = γV0Δt em (*), obtemos:  
γHg V0Hg Δt ‐ γvidro V0vidro Δt = V0vidro ‐ V0Hg 
1,8 . 10‐4 . 190Δt ‐ 1,2 . 10‐5 . 200Δt = 200 ‐ 190 
340 . 10‐4Δt ‐ 24 . 10‐4Δt = 10 
3,2 . 10‐2Δt = 10 ⇒ Δt = 310 °C 
 
11 Ao abastecer o carro em um posto de gasolina, você compra o combustível por volume e não por massa, isto é, você 
compra “tantos litros” e não “tantos quilogramas” de combustível. Assim, qual o melhor horário do dia para abastecer o 
carro se você quer fazer economia? 
SOLUÇÃO 
No período da manhã. A gasolina passou a noite esfriando, de manhã começará a ser aquecida. 
 
12  Uma  lâmina  bimetálica  é  constituída  por  uma  junção  de  duas  lâminas  retilíneas  que  têm  o  mesmo  comprimento 
quando  estão  à  temperatura  T.  Ao  aumentar  sua  temperatura  para  T  +  ΔT  a  lâmina  se  curva  formando  um  arco  de 
circunferência de espessura total d (figura abaixo). Supondo que os coeficientes de dilatação linear das lâminas sejam 
respectivamente  iguais  α1  e  α2,  com  α2  >  α1,  e  que  as  espessuras  de  cada  lâmina,  após  a  dilatação,  sejam  iguais.  O 
Professor Gomes pede que se deduza a expressão do raio de curvatura da junção entre as lâminas. 

 
SOLUÇÃO 
Suponha que as duas lâminas tenham, à temperatura T, o mesmo comprimento L0. Ao serem aquecidas, ambas sofrerão 
dilatação térmica e terão novos comprimentos dados por: L1 = L0∙(1+α1∙ΔT) e L2 = L0∙(1+α2∙ΔT). 
Como estão coladas uma na outra e como a lâmina 2 dilata mais do que a 1 (pois α2 > α1), a lâmina resultante irá se 
curvar em um arco de circunferência, cujo raio da lâmina externa é R2 e da lâmina interna é R1 (figura abaixo). 

 
Ambas subtendem o mesmo ângulo θ dado por: 

 
23 
L1 L 2
 
R1 R2
 
L2
R2  R1
L1
Como R2 = R1 + d/2, teremos 
d L 
R1   1 
2  L 2  L1 
 
d  1  1 T 
R1   
2  (2  1 )T 
O raio da junção é R = R1 + d/4. O que nos leva: 
d  2(1  1 T) 
R   1  
4  (2  1 )T 
Finalmente: 
d  2  (2  1 )T 
R   
4  (2  1 )T 
 
13 Com uma fita métrica de alumínio, uma haste de bronze é medida a 20 °C, obtendo um valor de 80 cm. Qual seria a 
leitura da medida obtida a 40 °C se a fita é precisa a 20 °C? αalumínio = 2,3.10‐5 °C ‐1; αbronze = 1,8.10‐5 °C‐1. 
SOLUÇÃO 
Primeiro, encontraremos o comprimento final da haste de bronze a partir dos dados, usando a relação L = L0(1 + αΔT).  
Lbronze = 80 [1 + 1,8.10‐5 (40 ‐ 20)] ⇒ Lbronze = 80,0288 cm 
Agora encontraremos o comprimento do "novo centímetro" na fita de alumínio. 
1 centímetro novo = 1 cm [1+ 2,3 10‐5 (40 ‐ 20)] = 1,00046 cm 
Finalmente, para descobrir o "comprimento aparente" da haste de bronze medida com o "novo centímetro", faremos 
uma divisão simples para encontrar quantas vezes este está contido no primeiro. 
Comprimento aparente = Lbronze/”novo centímetro” = 80,0288 cm/1,00046 cm 
Comprimento aparente = 79,992 
 
14 As hastes mostradas na figura têm os comprimentos L1 e L2 a uma temperatura de 0 °C. Determine o aumento de 
temperatura ΔT que o sistema deve sofrer para que as extremidades livres se toquem. 

 
SOLUÇÃO 
Irão se tocar quando: 
ΔL1 + ΔL2 = x 
logo, 
L1.α1.ΔT + L2.α2.ΔT = x 
x
T   
L1 1  L 2 2
 
15 Qual é a temperatura mais provável no fundo de um lago com gelo na superfície? Por que razão? 
SOLUÇÃO 
A temperatura mais provável no fundo de um lago é 4 °C pois a densidade da água é máxima nesta temperatura. 
 
16  Coloque  em  ordem  as  seguintes  substâncias,  em  termos  de  quantidade  de  calor  necessária  para  elevar  a  sua 
temperatura de 20 °C a 21 °C, do maior ao menor valor:  
i) um quilograma de mercúrio;  
ii) um quilograma de álcool etílico;  
iii) um mol de mercúrio;  
 
24 
iv) um mol de álcool etílico. 
SOLUÇÃO 
(ii),  (i),  (iv),  (iii).  Para  (i)  e  (ii),  a  grandeza  relevante  é  o  calor  específico  c  da  substância,  que  é  a  quantidade  de  calor 
necessária para elevar a temperatura de 1 quilograma dessa substância em 1 K. Conforme a tabela 3, esses valores são 
(i) 138 J para o mercúrio e (ii) 2428 J para o álcool etílico. Para (iii) e (iv), precisamos do calor específico molar C, que é a 
quantidade de calor necessária para elevar a temperatura de 1 mol dessa substância em 1 °C. Mais uma vez, conforme a 
tabela 3, esses valores são (iii) 27,7 J para o mercúrio e (iv) 111,9 J para o álcool etílico. (A razão dos calores específicos 
molares  é  diferente  dos  calores  específicos  porque  um  mol  de  mercúrio  e  um  mol  de  álcool  etílico  possuem  massas 
diferentes.) 
 
17 Consideremos uma amostra de alumínio, cujo calor específico é c = 0,217 cal/(g . °C). Se a massa da amostra é m = 50 
g, qual a quantidade de calor que deve trocar para que sua temperatura: 
a) aumente de 10 °C para 50 °C? 
b) diminua de 80 °C para 20 °C? 
SOLUÇÃO 
a)  Para  aumentar  a  temperatura  de  Ti  =  10  °C  para  Tf  =  50  °C,  a  amostra  deve  receber  calor,  sendo  a  variação  de 
temperatura ΔT = Tf ‐ Ti = 50 °C ‐ 10 °C = 40 °C. Utilizando a equação fundamental da calorimetria: Q = m . c . ΔT  
Q = 50 . 0,217 . 40 ⇒ Q = 434 cal 
b)  Para  que  a  temperatura  diminua  de  Ti  =  80  °C  para  Tf  =  20  °C,  a  amostra  deve  perder  calor,  sendo  a  variação  de 
temperatura ΔT = Tf ‐ Ti  = 20 °C ‐ 80 °C = ‐60 °C. Aplicando a equação: Q = m . c . ΔT 
Q = 50 . 0,217 . (‐60)  ⇒ Q = ‐651 cal 
 
18 Um recipiente adiabático contém 500 g de água, inicialmente a 20 °C. O conjunto é aquecido até 80 °C, utilizando‐se 
uma  fonte  de  calor  que  desenvolve  uma  potência  útil  de  200  W.  Considerando  o  calor  específico  da  água  igual  a  1,0 
cal/g °C e fazendo 1 cal igual a 4 J, quanto tempo foi gasto nesse aquecimento? 
SOLUÇÃO 
Pot Δt = m c ΔT 
200/4. Δt = 500 ∙ 1,0 ∙ (80 – 20) 
Δt = 600 s = 10 min 
 
19 Em Fortaleza, um fogão a gás natural é utilizado para ferver 2,0 litros de água que estão a uma temperatura inicial de 
19 °C. Sabendo que o calor de combustão do gás é de 12 000 cal/g, que 25% desse calor é perdido para o ambiente, que 
o calor específico da água vale 1,0 cal/g °C e que a densidade absoluta da água é igual a 1,0 g/cm3, que massa mínima de 
gás foi consumida no processo? 
SOLUÇÃO 
Q = m c ΔT 
12 000 ∙ mg ∙ 0,75 = 2 000 ∙ 1,0 ∙ (100 – 19) 
9 000 mg = 162 000 
mg = 18 g 
 
20 O Professor Gomes toma seu café‐da‐manhã em uma xícara de alumínio. A xícara possui uma massa igual a 0,120 kg 
e estava inicialmente a 20 °C quando o Professor Gomes a encheu com 0,300 kg de um café que estava inicialmente a 
uma  temperatura  de  70  °C.  Qual  é  a  temperatura  final  depois  que  o  café  e  a  xícara  atingem  o  equilíbrio  térmico? 
(Suponha  que  o  calor  específico  do  café  seja  igual  ao  da  água,  e que  não  exista  nenhuma  troca  de  calor  com  o  meio 
ambiente.) 
SOLUÇÃO 
Usando a tabela 3, o calor perdido (negativo) pelo café é dado por 
Qcafé = mcafé cágua ΔTcafé = (0,300 kg)(4190 J/kg ∙ K)(T − 70 °C) 
O calor ganho (positivo) pela xícara de alumínio é dado por 
Qalumínio = malumínio calumínio ΔTalumínio = (0,120 kg)(910 J/kg ∙ K)(T − 20 °C) 
A soma dessas duas quantidades de calor deve ser igual a zero, obtendo‐se a seguinte equação algébrica para T: 
Qcafé + Qalumínio = 0, 
(0,300 kg)(4190 J/kg ∙ K)(T − 70 °C) + (0,120 kg)(910 J/kg ∙ K)(T − 20 °C) = 0 
T = 66 °C. 
 

 
25 
21 Num recipiente termicamente isolado e com capacidade térmica desprezível, misturam‐se 200 g de água a 10 °C com 
um bloco de ferro de 500 g a 140 °C. Qual a temperatura final de equilíbrio térmico? 
Dados: calor específico da água = 1,0 cal/g °C; 
calor específico do ferro = 0,12 cal/g °C. 
SOLUÇÃO 
Como o recipiente tem capacidade térmica desprezível, ele não participa das trocas de calor. E, como é termicamente 
isolado, é correto afirmar que: 
Qferro + Qágua = 0 
Uma vez que o calor trocado é sensível, temos: 
(m c ΔT)ferro + (m c ΔT)água = 0 
500 ∙ 0,12(TE – 140) + 200 ∙ 1,0(TE – 10) = 0 
60(TE – 140) + 200(TE – 10) = 0 
60TE – 8 400 + 200TE – 2 000 = 0 
260TE = 10 400 ⇒ TE = 40 °C 
 
22 Num recipiente de capacidade térmica desprezível e termicamente isolado, são colocados 20 g de água a 60 °C e 100 
g  de  lascas  de  alumínio  a 40  °C.  O  equilíbrio  térmico  ocorre  à  temperatura  de  50  °C.  Qual  o  valor  do  calor  específico 
sensível do alumínio? 
Dado: calor específico da água = 1 cal/g °C 
SOLUÇÃO 
Qcedido + Qrecebido = 0 
(m c ΔT)água + (m c ΔT)alumínio = 0 
20 ∙ 1 ∙ (50 – 60) + 100 ∙ cAl ∙ (50 – 40) = 0 
– 200 + 1 000cAl = 0  ⇒ cAl = 0,20 cal/g °C 
 
23 Num recipiente de capacidade térmica desprezível, encontramos um líquido a 20 °C. Misturando 600 g de água a 80 
°C com esse líquido, obtemos uma temperatura de equilíbrio térmico igual a 60 °C. Qual o equivalente em água desse 
líquido? 
SOLUÇÃO 
Qcedido + Qrecebido = 0 
(m c ΔT)líquido + (m c ΔT)água = 0 
E ∙ 1,0 ∙ (60 – 20) + 600 ∙ 1,0 (60 – 80) = 0 
40E – 12 000 = 0 ⇒ E = 300 g 
 
24  Um  anel  metálico  de  massa  150  g,  inicialmente  à  temperatura  de  160  °C,  foi  colocado  em  uma  cavidade  feita  na 
parte superior de um grande bloco de gelo em fusão, como mostrado na figura. 

 
Após o equilíbrio térmico ser atingido, verificou‐se que 30 cm3 de gelo se fundiram. Considerando o sistema (gelo‐anel) 
termicamente isolado, qual o valor do calor específico do metal que constitui o anel? 
Dados: calor latente de fusão do gelo: 80 cal/g; densidade do gelo: 0,92 g/cm3. 
SOLUÇÃO 
Qcedido + Qrecebido = 0 
(m c ΔT)anel + (m LF)gelo = 0 
(m c ΔT)anel + (d V LF)gelo = 0 
150 ∙ ca (0 – 160) + 0,92 ∙ 30 ∙ 80 = 0 
–24 000 ca + 2 208 = 0  ⇒ ca = 0,092 cal/g °C 
 
25 Num calorímetro ideal, misturam‐se 200 g de gelo a 0 °C com 200 g de água a 40 °C. 
Dados: calor específico da água = 1,0 cal/g °C; 
calor latente de fusão do gelo = 80 cal/g. 
 
26 
Determine: 
a) a temperatura final de equilíbrio térmico da mistura; 
b) a massa de gelo que se funde. 
SOLUÇÃO 
a) Resfriamento da água até 0 °C: 
Q = m c ΔT = 200 ∙ 1,0 ∙ (0 – 40) 
Q = –8 000 cal (negativo porque é calor cedido) 
Fusão total do gelo: 
Q = m L = 200 ∙ 80 
Q = 16 000 cal 
Como a energia liberada pela água é menor que a necessária para a fusão total do gelo, a temperatura de equilíbrio será 
0 °C, restando gelo. 
T1 = 0 °C 
b) Q = m L  ⇒ 8 000 = m ∙ 80  ⇒ m = 100 g 
 
26  Num  copo  de  capacidade  térmica  desprezível,  tem‐se  inicialmente  170  cm3  de  água  a  20  °C.  Para  resfriar  a  água, 
colocam‐se  algumas  “pedras”  de  gelo,  de  massa  total  100  g,  com  temperatura  de  –20  °C.  Desprezando  as  perdas  de 
calor com o ambiente e sabendo que após um intervalo de tempo há o equilíbrio térmico entre a água líquida e o gelo, 
determine a massa de gelo remanescente. 
Dados: ρágua (densidade da água) = 1,0 g/cm3 
cágua (calor específico da água) = 1,0 cal/(g °C) 
LF (calor latente de fusão do gelo) = 80 cal/g 
cgelo (calor específico do gelo) = 0,5 cal/(g °C) 
SOLUÇÃO 
Qcedido + Qrecebido = 0 
(m c ΔT)água + (m c ΔT + m LF)gelo = 0 
(d V c ΔT)água + (m c ΔT + m LF)gelo = 0 
1,0 ∙ 170 ∙ 1,0 ∙ (0 – 20) + 100 ∙ 0,5 ∙ [0 – (–20)] + m ∙ 80 = 0 
–3 400 + 1 000 + 80 m = 0 
80 m = 2 400 ⇒ m = 30 g 
Restando no copo: 
mg = (100 – 30) g  ⇒ mg = 70 g 
 
27 O calor específico a pressão constante de um material é constante e igual a c. Uma quantidade do material, de massa 
m1  e  à  temperatura  T1  é  posta  em  contato  com  outra  quantidade  do  mesmo  material,  sendo  a  massa  m2  e  a 
temperatura T2. Deduza uma expressão geral para o cálculo da temperatura final de equilíbrio.  
SOLUÇÃO 
Não  há  variação  de  energia  interna,  porque  os  dois  corpos  constituem  um  sistema  isolado.  O  sistema  também  não 
interage mecanicamente com o exterior, ou seja, não há realização de trabalho. Portanto, de acordo com o princípio da 
conservação da energia, podemos escrever: 
Calor ganho = Calor perdido 
Estamos supondo que T2 seja maior que T1. Assim, a temperatura final (Tf) estará compreendida entre estes dois valores. 
O calor ganho pelo corpo frio será: 
Q1 = m1.c.(Tf – T1) 
O calor perdido pelo corpo quente será: 
 Q2 = m2.c.(T2 – Tf) 
Como Q1 = Q2, obtém‐se a temperatura de equilíbrio: 
m T  m2 T2
Tf  1 1  
m1  m2
 
28 A capacidade calorifica de um sistema é dada em função da temperatura pela relação: 
C' = AT‐ BT2 + (C/T) 
onde  A,  B,  C  são  constantes  e  T  é  a  temperatura.  Determine  a  expressão  do  calor  fornecido  ao  sistema  quando  a 
temperatura passar de um valor inicial Ti até um valor final Tf. 
SOLUÇÃO 
O calor fornecido ao sistema é obtido pela integral: 
 
27 
 
Q  C'dT  (AT  BT2  CT 1 )dT  

Integrando‐se a relação acima nos limites considerados, tem‐se: 
Q  A(Tf2  Ti2 ) / 2  B(Tf3  Ti3 ) / 3  Cln(Tf / Ti )  

29 Para intervalos de temperaturas entre 5°C e 50 °C, o calor especifico (c) de uma determinada substância varia com a 
temperatura  (t)  de  acordo  com  a  equação  c  =  1/60.t  +  2/15,  onde  c  é  dado  em  cal/g°C  e  t  em  °C.  Determine  a 
quantidade de calor necessária para aquecer 60 g desta substância de 10°C até 22°C. 
SOLUÇÃO 
Vamos reescrever o calor específico como uma função da temperatura: 
t8
c  
60
Percebemos que para cada valor de temperatura temos um calor específico diferente, assim: 
▪ t = 10 °C ⇒ c = 18/60 cal/g °C 
▪ t = 22 °C ⇒ c = 30/60 cal/g °C 
A área sob o gráfico “calor específico versus temperatura” nos fornece a razão Q/m. 


A c.T 
Q
m
 18 30 
 60  60  .12 Q
    
2 60
Q  288 cal

 

30 Duas esferas isotrópicas perfeitas de volume V0 estão sendo utilizadas em um experimento de termodinâmica. Uma 
delas, A, está suspensa por um fio ideal e outra, B, está sobre um apoio horizontal ideal.  As dissipações térmicas são 
desprezadas,  sendo,  portanto,  isolantes  térmicos  o  fio  e  o  apoio.  Nesse  cenário  é  cedida  aos  corpos  A  e  B  uma 
quantidade de calor sensível igual para ambos, o que acarreta uma variação de altura do centro de massa das esferas 
igual a h. A aceleração da gravidade é g e o calor específico das esferas é c. A temperatura inicial das esferas é 0 °C. O 
Professor Gomes pede que se determine a diferença de temperatura das esferas após o aquecimento. 

 
SOLUÇÃO 
Como ambas são isotrópicas perfeitas de volume V0, elas irão se dilatar igualmente. Seja R o raio das esferas. Adote o 
referencial no plano horizontal em que a esfera B está apoiada. Com isso o CM da esfera B está a uma distância R acima 
do plano. Quando for fornecida uma quantidade de calor Q a esfera, ela irá se dilatar e seu CM sofrerá uma variação de 
h. A posição do novo CM será: R + h. 
Houve uma variação de: mgh na energia potencial gravitacional da esfera. Essa variação foi por conta da quantidade Q = 
m.c.ΔTB de calor recebida. Então: 
m.c.ΔTB = mgh  ⇒ ΔTB = gh/c. ⇒ TB ‐ TBo = gh/c  ⇒ TB = gh/c 
Como  se  trata  de  um  fio  ideal,  o  centro  de  massa  da  esfera  A  descerá.  Ficara  numa  posição  R  +  h  abaixo  da  linha 
horizontal,  com  isso  a  variação  da  energia  gravitacional  será  de  ‐mgh.  Essa  variação  foi  por  conta  da  quantidade  Q  = 
m.c.ΔTA de calor recebida. 

 
28 
Então: 
‐mgh = m.c.ΔTA  ⇒ ΔTA  = ‐gh/c  ⇒ TA ‐ TAo = ‐gh/c  ⇒ TA = ‐gh/c 
Por fim: 
TB ‐ TA = 2gh/c 
 
31 Considere uma garrafa térmica fechada contendo uma certa quantidade de água inicialmente a 20 °C. Elevando‐se a 
garrafa a uma certa altura e baixando‐a em seguida, suponha que toda a água sofra uma queda livre de 42 cm em seu 
interior. Este processo se repete 100 vezes por minuto. Supondo que toda a energia cinética se transforme em calor a 
cada movimento, determine o tempo necessário para ferver toda a água. 
SOLUÇÃO 
n.m.g.h 100.m.10.0,42
Pot  
t 60  
Pot  7m W
Logo, para que toda a água ferva temos:  
Q = m.c.ΔT = Pot.Δt 
m.4,2.103.80 = 7m.Δt 
Δt = 48000 s 
 
32  Uma  caixa  de  isopor  usada  para  manter  as  bebidas  frias  em  um  piquenique  (figura  abaixo)  possui  área  total 
(incluindo a tampa) igual a 0,80 m2, e a espessura de sua parede mede 2,0 cm.  

 
A caixa está cheia de água, gelo e latas de Coca‐Cola a 0 °C. Qual é a taxa de fluxo de calor para o interior da caixa, se a 
temperatura da parede externa for 30 °C? Qual é a quantidade de gelo que se liquefaz durante um dia? 
SOLUÇÃO 
Supomos  que  o  fluxo  de  calor  seja  aproximadamente  o  mesmo  que  ocorreria  através  de  uma  placa  com  área  igual  a 
0,80 m2 e espessura igual a 2,0 cm = 0,020 m (figura a seguir).  

   
Achamos o valor de k na tabela 5. A taxa de transferência de calor (ou taxa de fluxo de calor) é dada por 
TH  TC 30  0
H  kA  (0,010)(0,80)  12 W  
L 0,020
O fluxo total de calor Q em um dia (86400 s) é dado por 
Q = Ht = (12 J/s)(86400 s) = 1,04.106 J 
O calor de fusão do gelo é igual a 3,34.105 J/kg, portanto a quantidade de gelo fundida por essa quantidade de calor é 
Q 1,04.106
m   3,1 kg  
L f 3,34.10 5
 

 
29 
33  Uma  barra  de  aço  de  10,0  cm  de  comprimento  é  soldada  pela  extremidade  a  uma  barra  de  cobre  de  20,0  cm  de 
comprimento.  As  duas  barras  são  perfeitamente  isoladas  em  suas  partes  laterais.  A  seção  reta  das  duas  barras  é  um 
quadrado de lado igual a 2,0 cm. A extremidade livre da barra de aço é mantida a 100 °C pelo contato com vapor d’água 
obtido por ebulição, e a extremidade livre da barra de cobre é mantida a 0 °C por estar em contato com gelo. Calcule a 
temperatura na junção entre as duas barras e a taxa total da transferência de calor. 
SOLUÇÃO 
A figura abaixo mostra a situação. Usamos a equação do fluxo de calor duas vezes, uma para cada barra, e igualamos as 
duas  equações  de  transferência  de  calor  Hcobre  e  Haço.  Ambas  as  equações  de  transferência  de  calor  envolvem  a 
temperatura T na junção, e essa é uma de nossas incógnitas. 

 
Igualando as duas expressões, obtemos: 
k aço .A.(100  T) k cobre .A.(T  0)
Haço   Hcobre    
L aço L cobre
A área A é a mesma e pode ser cancelada nas duas equações. Substituindo Laço = 0,100 m, Lcobre = 0,200 m, e usando os 
valores de k da tabela 5, achamos 
(50,2).(100  T) (385).(T  0)
  
0,100 0,200
Reagrupando e explicitando T, encontramos T = 20,7 °C. 
Podemos calcular a taxa total de transferência de calor substituindo T em qualquer uma das relações anteriores: 
(50,2).(0,0200)2 .(100  20,7)
Haço   15,9 W  
0,100
 
34  Três  reservatórios  térmicos  estão  dispostos  conforme  ilustra  a  figura  seguinte.  Os  reservatórios  A  e  B  estão 
permanentemente  a  0  °C  e  120  °C,  respectivamente.  O  reservatório  C  contém  água  e  vai  perdendo  calor  para  o 
ambiente,  mantendo‐se  a  29,5  °C.  As  barras  têm  seção  transversal  de  9  cm2,  condutibilidade  2,0  cal/(cm.s.°C),  os 
comprimentos  indicados  na  figura  e  são  isoladas  termicamente  do  ambiente.  Qual  será  a  temperatura  na  junção  das 
barras? 

 
SOLUÇÃO 
Denominando‐se por J o ponto de junção das barras, podemos observar que o fluxo de calor de B para J é a soma dos 
fluxos de J para C com J para A, assim, como 
kA
  (T  T')  temos: 
L
BJ   JC   JA  
logo, 
k.A.(120  T) k.A.(T  0) k.A.(T  29,5)
  , resolvendo obtemos: 
3L 2L L
5 5 2
T = 42 °C 
 
35 Duas hastes do mesmo comprimento e material transferem uma determinada quantidade de calor em 12 segundos 
quando  unidas  em  série.  Em  quanto  tempo  transferirão  a  mesma  quantidade  de  calor,  nas  mesmas  condições  de 
temperatura, quando são unidos em paralelo?  
SOLUÇÃO 
 
30 
Seja  Q  o  calor  transferido.  Se  k  é  o  coeficiente  de  condutibilidade  térmica  de  cada  haste,  então  o  coeficiente  de 
condutibilidade térmica quando associadas em série é 2k. Se t1 é o tempo de transferência de calor, teremos: 
2k.A.T.t1
Q1   
L
O  coeficiente  de  condutibilidade  térmica  quando  associadas  em  paralelo  é  k/2.  Se  t2  é  o  tempo  de  transferência  de 
calor, teremos: 
k
.A.T.t2
Q2  2  
L
Como Q1 = Q2  
k
.A.T.t2
2k.A.T.t1 2
  
L L
t2 = 4t1  
t2 = 48 s 
 
36  Uma  placa  quadrada  de  aço,  com  lado  igual  a  10  cm,  é  aquecida  em  uma  forja  até  uma  temperatura  de  800  °C. 
Sabendo que a emissividade é igual a 0,60, qual é a taxa total de energia transmitida por radiação? 
SOLUÇÃO 
Usamos  a  equação  H  =  AeσT4  para  calcular  H  a  partir  dos  valores  dados.  A  área  total  da  superfície,  incluindo  os  dois 
lados da placa, é 2(0,10m)2 = 0,020 m2. Devemos converter a temperatura na escala Kelvin: 800 °C = 1073 K. Então, a 
equação H = AeσT4 fornece 
H = Ae σT4 = (0,020 m2) (0,60) (5,67.10–8 W/ m2 . K4) (1073 K)4 = 900 W 
 
37 Sabendo que a área total do corpo é 1,20 m2, e que a temperatura da superfície é 30 °C = 303 K, calcule a taxa total 
de transferência de calor do corpo por radiação. Se o meio ambiente está a uma temperatura de 20 °C, qual é a taxa 
resultante  do  calor  perdido  pelo  corpo  por  radiação?  A  emissividade  do  corpo  é  próxima  da  unidade, 
independentemente da pigmentação da pele. 
SOLUÇÃO 
A taxa de radiação da energia que o corpo emite é dada pela equação H = AeσT4, e a taxa total de perda de calor é dada 
pela equação Htotal = Aeσ(T4 – Ts4). Fazendo e = 1 na equação H = AeσT4 , descobrimos que o corpo irradia a uma taxa 
H = AeσT4 = (1,20 m2) (1) (5,67.10–8 W/m2∙K4) (303 K)4 = 574 W 
Essa  perda  é  parcialmente  compensada  pela  absorção  da  radiação,  que  depende  da  temperatura  ambiente.  A  taxa 
resultante da transferência de energia é dada pela relação Htotal = Aeσ(T4 – Ts4): 
Htotal = Aeσ(T4 – Ts4) = (1,20 m2) (1) (5,67.10–8 W/m2 ∙ K4)× [(303 K)4 − (293 K)4] = 72 W 
 
EXERCÍCIOS PARA RESOLVER 
 
01 Explique por que não faria sentido usar um termômetro de vidro de tamanho normal para medir a temperatura de 
uma gota de água quente. 
 
02  A  temperatura  de  um  corpo  é  medida  simultaneamente  nas  escalas  Celsius  e  Fahrenheit.  Como  resultado,  a 
temperatura  na  escala  Celsius  é  o  dobro  da  outra.  Qual  o  valor  aproximado  da  temperatura  do  corpo  na  escala 
Fahrenheit? 
 
03 Comparando‐se a escala E de um termômetro com a escala Celsius, obteve‐se o gráfico da figura de correspondência 
entre as medidas.  

 
 
31 
Quando o termômetro Celsius estiver registrando 60 °C, qual o valor marcando no termômetro E? 
 
04 Um termômetro defeituoso, graduado na escala Fahrenheit indica 30 °F para a temperatura de fusão do gelo e 214 °F 
para a temperatura de ebulição da água. Nesse termômetro, determine a única temperatura medida corretamente. 
 
05  Um  médico  inglês  mede  a  temperatura  de  um  paciente  com  suspeita  de  infecção  e  obtém  em  seu  termômetro 
clínico o valor de 102,2 °F (graus Fahrenheit). 
a) Tem ele motivo de preocupação com o paciente? Justifique. 
b) Por que um doente com febre sente frio? Responda e defina também o conceito físico de calor. 
 
06 Dois corpos A e B estavam inicialmente à mesma temperatura. Os dois corpos são, então, aquecidos de modo que a 
temperatura de A sofre um aumento de 1 °C e a temperatura de B sofre um aumento de 1 °F. No final, qual dos dois 
está mais quente? 
 
07 Responda: 
a)  Quando  um  termômetro  de  mercúrio  tem  o  seu  bulbo  colocado  próximo  a  um  chama,  a  coluna  de  mercúrio 
inicialmente desce um pouco para depois subir. Por quê? 
b) Certa vez um jornal publicou a seguinte notícia: “Ontem os termômetros registraram a temperatura mais quente dos 
últimos dez anos” Nessa frase há um erro conceitual. Qual? 
c) Podemos atribuir uma temperatura ao vácuo? 
 
08 A relação entre uma certa escala termométrica A e a escala Celsius é A = C + 3 e entre uma escala termométrica B e a 
escala Fahrenheit é B = 2F – 10. Qual a relação entre as escalas A e B? 
 
09 É dado um termômetro X tal que 60 °X correspondem a 100 °C; 20 °X correspondem a 20 °C; 0 °X corresponde a 0 °C. 
As  leituras Celsius variam conforme trinômio de segundo grau nas leituras X. Deduzir a equação que dá leituras Celsius 
em função de leituras X. 
 
10 Um termômetro de gás registra uma pressão absoluta que corresponde a 325 mm de mercúrio quando em contato 
com a água  que está no  ponto triplo.  Qual seria a  pressão lida no termômetro se estivesse em  contato com água  no 
ponto de ebulição normal? 
 
11 A pressão de um gás no ponto triplo da água é 1,35 atm. Se o seu volume permanecer inalterado, qual será a sua 
pressão à temperatura em que o CO2 solidifica? 
 
12 Uma escala termométrica absoluta Q marca 160 °Q para ‐ 43 °C. Para uma substância que estava inicialmente a ‐16 
°F e sofreu um aquecimento de 80 °Q, qual será a sua temperatura final em °F? 
 
13  Se  tem  duas  escalas  termométricas  A  e  B,  de  modo  que  a  água  ferve  a  240  °A  e  180  °B.  Se  aumentarmos  a 
temperatura em 1 °A é equivalente a aumentar a temperatura em 1,5 °B, calcule a que temperatura as escalas A e B 
coincidem. 
 
14 O coeficiente de dilatação linear de um certo material é   = 18 . 10‐6 °C‐1. Se usássemos o grau Fahtenheit, qual seria 
o valor desse coeficiente? 
 
15 Um arame de aço, de  coeficiente de dilatação linear 1,2.10‐5  °C‐1, mantém‐se dobrado conforme a figura, com sua 
extremidade A engastada no teto, a uma temperatura de 25 °C. Quando a temperatura se eleva para 100 °C, como se 
move o ponto E (outra extremidade)? 

 
 

 
32 
16 A barra da figura é composta de dois segmentos: um de comprimento   e coeficiente de dilatação linear αA e outro 
de comprimento 2  e coeficiente de dilatação linear αB. Determine o coeficiente de dilatação linear dessa barra, α, em 
função de αA e αB. 

 
 
17 O comprimento de uma barra, medido com uma régua de ferro à temperatura ambiente de 20 °C, é de 20,05 cm. A 
barra e a régua são colocadas em um forno a 270 °C e a medida da barra com a régua é agora de 20,11 cm. Calcule o 
coeficiente de dilatação térmica do material da barra. 
 
18  Três  barras  retas  de  alumínio,  invar  e  aço,  de  mesmo  comprimento,  formam  a  20  °C  um  triângulo  equilátero  com 
articulações nos vértices. A que temperatura o ângulo oposto ao lado de invar será de 59,95°? 
 
19 Uma barra de cobre foi recurvada tomando a forma de uma semicircunferência. As extremidades foram unidas por 
uma outra barra reta constituída por dois metais: uma parte, de comprimento x, era de zinco e a outra, de comprimento 
y, de platina. 

 
São dados os coeficientes de dilatação lineares: 
cobre = 17 ∙ 10–6 °C–1; 
zinco = 29 ∙ 10–6 °C–1; 
platina = 9 ∙ 10–6 °C–1. 
Para que o arco de cobre conserve sua forma semicircular, a qualquer temperatura a que seja levado, qual deve ser a 
razão x/y entre os comprimentos iniciais x e y dos segmentos de zinco e platina? 
 
20 A figura mostra uma lâmina bimetálica, de comprimento L0 na temperatura T0, que deve tocar o contato C quando 
aquecida. A lâmina é feita dos metais I e II, cujas variações relativas do comprimento ΔL/L0 em função da variação de 
temperatura ΔT = T – T0 encontram‐se no gráfico. 

 
Determine: 
a) o coeficiente de dilatação linear dos metais I e II; 
b)  qual  dos  metais  deve  ser  utilizado  na  parte  superior  da  lâmina  para  que  o  dispositivo  funcione  como  desejado. 
Justifique sua resposta. 
 

 
33 
21 O sistema mostrado na figura abaixo é composto por dois fios do mesmo material e uma barra que, em conjunto, 
formam um quadrado com o teto, e todos à temperatura ambiente T0 = 20 °C. A que temperatura será observado que 
os fios formam com a barra um ângulo de 74°? Desprezar os efeitos de deformação por elasticidade. αfio  = 5.10‐4 °C‐1; 
αbarra = 5,88.10‐3 °C ‐1. 

   
 
22 Uma trena metálica de 5 m de comprimento é exata a 15 °C. Um dia, quando a temperatura ambiente é de 35 °C, é 
medido um terreno, com 100 m de comprimento. Qual é o verdadeiro comprimento do terreno, sabendo que αtrena  = 
4.10‐4 °C‐1? 
 
23  Considere  um  sólido  constituído  de  um  material  anisótropo.  Nestes  materiais  os  coeficientes  de  dilatação  linear 
dependem  geralmente  da  direção  de  observação.  Em  todos  os  cristais  é  possível  determinar  três  direções 
perpendiculares duas a duas, tais que, um cubo de cristal com arestas paralelas a essas direções, aquecido, conserva os 
ângulos retos, embora os comprimentos dos lados variem desigualmente. Tais direções constituem os eixos principais 
de dilatação e os coeficientes de dilatação linear relativos a essas direções são chamados coeficientes de dilatação linear 
principais.  Sejam  αx,  αy,  e  αz  os  coeficientes  de  dilatação  linear  principais  de  um  cristal.  Mostre  que  o  coeficiente  de 
dilatação volumétrica é dado por:  
γ = αx + αy + αz. 
 
24 Uma armação apresenta um formato retangular de lados a e b, sendo o lado a duas vezes maior do que o lado b, 
conforme a figura abaixo. Os coeficientes de dilatação linear dos lados a e b são iguais a αa e αb respectivamente. Ao 
longo da diagonal da armação retangular, é fixada uma barra de comprimento x feita de um certo material, com coe‐
ficiente de dilatação linear αx. 

 
Determine o coeficiente de dilatação linear αx em função dos coeficientes de dilatação αa e αb, de forma que a barra não 
fique nem tensionada e nem comprimida, devido às variações de temperatura. 
 
25  Uma  haste  de  comprimento  20  cm  feita  de  um  metal  A  se  expande  em  0,075  cm  quando  sua  temperatura  é 
aumentada de 0 °C a 100 °C. Outra haste de um metal diferente B com o mesmo comprimento se expande em 0,045 cm 
para a mesma mudança de temperatura. Uma terceira haste do mesmo comprimento é composta de duas partes, uma 
de metal A e outra de metal B. Esta haste se expande em 0,060 cm para a mesma mudança de temperatura. Determine 
o comprimento porção feita de metal A. 
 
26 O coeficiente de expansão linear de uma haste não homogênea muda linearmente de α1 a α2 de uma extremidade 
para a outra extremidade da haste. Determine o coeficiente de expansão linear efetivo da haste. 
 
27 Três hastes do mesmo comprimento estão dispostas formando um triângulo equilátero. Duas hastes são feitas  do 
mesmo material de coeficiente de expansão linear α1 e a terceira haste que forma a base do triângulo tem coeficiente 
de  expansão  α2∙  Determine  a  relação  α1/α2  para  que  a  altura  do  triângulo  permanece  a  mesma  em  todas  as 
temperaturas. 
 
 
34 
28 Um arame de coeficiente de dilatação linear α comprimento L é dobrada em forma de circunferência deixando uma 
abertura. Se aumentarmos a temperatura em ΔT, a abertura da circunferência aumentará, diminuirá ou ficará a mesma? 
 
29  Duas  barras  de  metal  sobrepostas  e  soldadas  por  uma  só  extremidade  apresentam  em  qualquer  temperatura  a 
mesma diferença de comprimento. Aquecendo ambas as barras de T °C, a relação dos seus comprimentos passa a ser: 
L1/L2 = n. Sabendo que seus coeficientes de dilatação são α1 e α2, encontrar "n" em função de T, α1 e α2. 
 
30 Um relógio de pêndulo simples é montado no pátio de um laboratório em Novosibirsk, na Sibéria, utilizando um fio 
de suspensão de coeficiente de dilatação 1.10–5  °C‐1. O pêndulo é calibrado para marcar a hora certa em um bonito dia 
de verão de 20 °C. Em um dos menos agradáveis dias do inverno, com a temperatura a ‐40 °C, o relógio:  
a) adianta ou atrasa?                
b) quanto tempo por dia.  
 
31  Um  relógio  de  pêndulo  é  calibrado  no  frio  inverno  siberiano,  onde  a  temperatura  média  diária  é  T0  =  ‐  30  °C.  O 
pêndulo é constituído por um fio de prata e tem comprimento L0 = 0,25 m. Considere que o período do pêndulo desse 
relógio é dado por P = 2π (L/g)1/2. O coeficiente de dilatação linear da prata é α = 2,0. 10‐5 °C‐1. No verão, a temperatura 
média  diária  no  local  chega  a  T  =  +  30  °C.  Calcule  quanto  o  relógio  atrasará  por  dia  devido  a  esta  variação  de 
temperatura. 
 
32 Um antigo relógio de pêndulo de aço (coeficiente de dilatação linear α = 10.10‐6 oC‐1) funciona corretamente durante 
o inverno. No verão, ele passa a atrasar o equivalente a 2 min por mês. Determine a diferença entre as temperaturas 
médias de verão e inverno. 
 
33 Um aluno do IFCE levou um relógio de pêndulo simples, de Fortaleza, no litoral cearence, para São josé dos Campos, 
a 600m acima do nível do mar. O relógio marcava a hora correta em Fortaleza, mas demonstra uma pequena diferença 
em São josé. Considerando a Terra como uma esfera com seu raio correspondendo ao nível do mar. Em São José dos 
Campos o relógio 
a) Adianta ou Atrasa? 
b) Quanto tempo por dia? 
 
34 Um relógio de pêndulo de metal adianta t1  = 3 s por dia a uma temperatura T1  = 5 °C e atrasa t2  = 30 s por dia a uma 
temperatura T2 = 30 °C. Com esses dados, calcule o coeficiente de expansão linear do metal do pêndulo. 
 
35 Um pêndulo de comprimento L tem um período T. Qual o novo período se a sua temperatura é aumentada por X °C? 
O coeficiente de dilatação linear é α. 
 
36 A temperatura de uma moeda de cobre aumenta de 100 °C e seu diâmetro cresce 0,18%. Dê o aumento percentual, 
com  dois  algarismos  significativos,  (a)  na  área,  (b)  na  espessura  (c)  no  volume  e  (d)  na  massa  da  moeda,  (e)  Qual  o 
coeficiente de dilatação linear da moeda? 
 
37  Um  fio  circular  envolve  completamente  uma  moeda  de  50  cm  de  raio.  Se  houver  um  aumento  de  100  °C  na 
temperatura do sistema, que separação existirá entre o fio e a moeda? αfio = 3.10‐5 °C‐1, αmoeda = 1.10‐5 °C‐1. 
 
38  Uma  haste  de  metal  tem  um  comprimento  L0  e  uma  secção  S0  a  0  °C.  Variando  a  temperatura,  sua  seção  é 
aumentada em 6%. Calcule a variação percentual que o seu comprimento experimenta no final do processo. 
 
39  O  tanque  de  gasolina  de  um  automóvel  é  abastecido  com  97%  de  sua  capacidade  total,  VT.  O  processo  de 
abastecimento foi realizado a uma temperatura de 0  oC. O carro foi transportado por uma carreta até uma localidade 
onde  a  temperatura  é  de  40  oC.  Com  o  aumento  da  temperatura,  poderá  acorrer  um  derramamento  de  combustível, 
devido à expansão deste provocada pela variação de temperatura. Considere que os coeficientes de expansão térmica 
da gasolina e do tanque são respectivamente VG = 9.10‐4 oC‐1 e VT = 1.10‐5 oC‐1 
a) Qual será o aumento de volume do combustível provocado pelo aumento de temperatura? 
b) Verifique se a gasolina irá derramar ou não para fora do tanque, considerando a situação descrita anteriormente. 
 

 
35 
40 Um vaso de vidro pyrex (αv = 10‐5 °C‐1) tem um volume Vv = 1080 cm3 à temperatura To = 10 °C. A esta temperatura é 
introduzido  um  volume  VL  =  1006  cm3  de  um  líquido  (γL  =  4.10‐4  °C‐1).  A  que  temperatura  máxima  o  sistema  deve  ser 
aquecido sem derramamento de líquido? 
 
41 Se tem uma esfera oca de raio (R) e em seu interior outra esfera de raio (r). Determine a razão (R/r) entre os raios 
para que o volume da parte interna não varie com o aumento da temperatura do sistema. A relação dos coeficientes de 
dilatação linear das esferas é αr = 8 αR. 
 
42 Com base em seus conhecimentos de termodinâmica, como você explica o fato que, no deserto, as noites são muito 
frias, apesar do sol forte dos dias? 
 
43 Mediante chave seletora, um chuveiro elétrico tem a sua resistência graduada para dissipar 4,0 kW no inverno, 3,0 
kW no outono, 2,0 kW na primavera e 1,0 kW no verão. Numa manhã de inverno, com temperatura ambiente de 10 °C 
foram  usados  10,0    de  água  desse  chuveiro  para  preencher  os  16%  do  volume  faltante  do  aquário  de  peixes 
ornamentais,  de  modo  a  elevar  sua  temperatura  de  23  °C  para  28  °C.  Sabe‐se  que  20%  da  energia  é  perdida  no 
aquecimento do ar, a densidade da água é   = 1,0 g/cm3 e calor específico da água é 4,18 J/gK. Considerando que a água 
do chuveiro foi colhida de 10 minutos, em que posição se encontrava a chave seletora? Justifique. 
 
44 Em tempos de economia de energia elétrica, um estudante tem a seguinte ideia para esquentar a água de seu banho. 
A caixa d'água de sua casa tem forma cúbica, com volume de 1 m3, e encontra‐se cheia de água. O estudante imagina 
então  soltar  do  repouso,  a  partir  da  superfície  da  água,  esferas  de  massa  1  kg  cada.  A  energia  dissipada  devido  à 
viscosidade da água e à colisão das esferas com o fundo da caixa d'água pode, em princípio, elevar a temperatura da 
água.  Suponha  que  toda  a  energia  dissipada  seja  transferida  para  a  água  na  forma  de  calor  e  que  não  haja  perdas 
térmicas nesse processo. Considere ainda as esferas puntiformes e despreze o calor absorvido por elas. 
a) Calcule quantas esferas o estudante deverá soltar a fim de elevar em 1 °C a temperatura da água. 
b) Se o estudante soltar as esferas à taxa de uma esfera por segundo, calcule a ordem de grandeza do número de horas 
que ele levará nesse processo. 
 
45  Uma  bala  de  chumbo,  deslocando‐se  com  velocidade  v,  colide  com  uma  placa  de  aço  e  pára.  Se  a  colisão  é 
totalmente  inelástica,  e  a  placa  de  aço  permanece  à  temperatura  inicial,  o  Professor  Gomes  pede  que  você  calcule  o 
valor mínimo da velocidade v para que a bala seja totalmente liquefeita. 
Considere os dados: 
To = 30 °C é a temperatura da bala e da placa, imediatamente antes da colisão.  
T = 330 °C é à temperatura de fusão do chumbo.  
cPb = 120 J/(kg°C) é o calor específico do chumbo.  
Lf = 28.800 J/kg é o calor de fusão do chumbo.  
 
46 Uma bola de massa m, cuja velocidade inicial é Vi = 20 m/s, sofre a ação de uma força aceleradora constante de 15 N, 
durante  um  percurso  retilíneo  de  10  m.  Ao  final  do  percurso  a  bola  se  choca  inelasticamente  com  uma  parede 
produzindo,  entre  outros  efeitos,  deformação  e  calor.  Suponha  que  apenas  50%  da  energia  cinética  da  bola  seja 
convertida  em  calor  e  que  75%  deste  calor  seja  absorvido  pela  bola.  Se  o  calor  específico  da  bola  vale  0,2  J/g  °C  e  o 
aumento de temperatura da bola foi de 6 °C, qual é a massa da bola?  
 
47 O Professor Gomes conta que um vaporizador contínuo possui um bico pelo qual entra água a 20 °C de tal maneira 
que  o  nível  de  água  no  vaporizador  permanece  constante.  O  vaporizador  utiliza  800  W  de  potência,  consumida  no 
aquecimento da água até 100 °C e na sua vaporização a 100 °C. Determine a vazão de água pelo bico em ml/s. 
Dados:  
calor específico sensível da água = 4,18.103 J/kg C 
densidade absoluta da água = 1000 kg/m3 
calor específico latente de vaporização da água =2,26.106 J/kg. 
 
48 Misturam‐se 60 gramas de água a 20 °C com 80 g de gelo a 0 °C. Supondo que só há trocas de calor entre a água e o 
gelo, calcule a massa final de líquido. Dados: calor de fusão do gelo = 80 cal/g, calor específico da água = 1,0 cal/g.°C. 
 

 
36 
49 Considere um calorímelro no qual existe uma certa massa de liquido. Para aquecer o conjunto liquido‐calorímetro de 
30 °C para 60 °C são necessários Q1J. Por outro lado, Q2J elevam de 40 °C para 80 °C o calorímetro juntamente com o 
triplo da massa do líquido. 
a) Determine a capacidade térmica do calorímetro nas seguintes situações:  
i) Q1 = 2000 J. Q2 = 4000 J 
ii) Q1 = 2000 J. Q2 = 7992 J 
b)  Com  base  nestes  dados,  em  qual  das  duas  situações  a  influência  do  material  do  calorímetro  pode  ser 
desconsiderada? Justifique sua conclusão. 
 
50 Dentro de um calorímetro de capacidade térmica 50 J °C–1 deixa‐se cair um sistema de duas massas de 100 g cada 
uma, ligadas por uma mola de massa desprezível. A altura da qual o sistema é abandonado é de 1,0 m acima do fundo 
do calorímetro e a energia total de oscilação do sistema é, inicialmente, de 1,5 J. Dada a aceleração da gravidade g = 10 
m.s–2  e  sabendo  que  após  um  certo  tempo  as  duas  massas  se  encontram  em  repouso  no  fundo  do  calorímetro, 
determine  a  variação  da  temperatura,  no  interior  do  calorímetro,  desprezando‐se  a  capacidade  térmica  do  sistema 
oscilante. 
 

 
 
51 Uma geladeira de 280 litros de volume interno é aberta, em média, 20 vezes num dia. Durante o intervalo de tempo 
em que a geladeira permanece aberta, toda a massa de ar contida em seu interior é espontaneamente substituída por 
parte do ar existente no local onde ela se encontra. A variação de temperatura que a massa de ar sofre no interior da 
geladeira, a cada vez que a porta é aberta e fechada, é de 25 °C. A densidade média e o calor específico do ar são dados 
por d = 0,0012 g/cm3 e c = 0,24 cal/g °C, respectivamente, com 1 cal = 4,18 J. 
a)  Qual  é  o  processo  responsável  pela  movimentação  do  ar  do  interior  para  o  exterior  da  geladeira  e  vice  versa? 
Explique a sua resposta. 
b) Quantos quilowatts‐hora são retirados pela geladeira do ar durante o período de um mês? 
 
52 A humanidade atingiu, em outubro, a cifra de 6.109 habitantes. Supondo que o consumo médio, por habitante, de 
energia calorífica proveniente dos alimentos é da ordem de 2000 kcal (1 kcal = 1000 cal) e que o calor de fusão do gelo 
vale L = 80 cal/g, calcule: 
a) quantas toneladas de gelo seriam transformadas em água, diariamente, caso essa energia fosse utilizada para fundir 
gelo a 0 °C (a densidade do gelo é de 0,9 g/cm3); 
b) quais seriam, aproximadamente, as dimensões de um bloco de gelo de forma cúbica que seria derretido? 
 
53 Um bloco de gelo com 725 g de massa é colocado num calorímetro contendo 2,50 kg de água a uma temperatura de 
5,0 °C, verificando‐se um aumento de 64 g na massa desse bloco, uma vez alcançado o equilíbrio térmico. Considere o 
calor específico da água (c = 1,0 cal/g °C) o dobro do calor específico do gelo, e o calor latente de fusão do gelo de 80 
cal/g. Desconsiderando a capacidade térmica do calorímetro e a troca de calor com o exterior, determine a temperatura 
inicial do gelo. 
 
54 No instante t = 0, um aquecedor de 400 W é colocado num pote contendo 2,0 litros de água a 10 °C. Após 7 minutos 
coloca‐se, no mesmo pote, um outro aquecedor idêntico para acelerar o aquecimento. Quanto tempo levará, a partir de 
t  =  0,  para  que  a  água  atinja  a  temperatura  de  90  °C,  supondo  que  apenas  80%  da  energia  elétrica  fornecida  pelos 
aquecedores seja absorvida pela água? Considere o calor específico da água como ca = 4200 J/kg °C.  
 
55 Um projétil é feito de um material que muda de cor conforme a temperatura em que se encontra (ver tabela). 
 
(°C)  0 e 10  10 a 20  20 a 30  30 a 40  40 a 50  50 a 60 
cor  amarelo  verde  azul  violeta  marrom  preto 
 

 
37 
Ele é impulsionado sobre um plano horizontal com uma velocidade de 120 m/s e corre sobre ele até parar por efeito do 
atrito.  Sabendo  que  a  temperatura  ambiente,  no  instante  em  que  o  corpo  foi  impulsionado,  era  12  °C  e  que  o  calor 
específico sensível do material de que ele é feito vale 300J/(kg .°C), determine quais as cores que o projétil apresentou 
durante  o  movimento.  Supor  que  toda  a  energia  gerada  pelo  atrito  seja  transformada  em  térmica  e  absorvida  pelo 
projétil. 
 
56 a) Dois cubos de gelo de 50 g são colocados em um recipiente de vidro contendo 200 g de água. Se a água estava 
inicialmente à temperatura de 25 °C e se o gelo veio diretamente do freezer a –15 °C qual será a temperatura final do 
sistema quando a água e o gelo atingirem a mesma temperatura?  
b) Supondo que somente um cubo de gelo foi usado em (a), qual a temperatura final do sistema? Ignore a capacidade 
térmica do vidro. 
 
57 O Professor Gomes conta que dois blocos de metal são isolados de seu ambiente. O primeiro bloco que tem massa 
m1 = 3,16 kg e temperatura inicial T1 = 17 °C tem um calor específico quatro vezes maior do que o segundo bloco. Este 
está à temperatura T2 = 47 °C e seu coeficiente de dilatação linear é 15.10–6 °C–1. Quando os dois blocos são colocados 
juntos e alcançam seu equilíbrio térmico, a área de uma face do segundo bloco diminui em 0,03%. Encontre a massa 
deste bloco. 
 
58  Uma  esfera  metálica  de  massa  m1,  calor  específico  c1  e  coeficiente  de  dilatação  linear  α,  tem  raio  r0  a  uma 
temperatura T1. Tal esfera é imersa em um líquido de massa m2 e calor específico c2, que se encontra a temperatura T2 > 
T1. Supondo que o recipiente que contém o líquido está isolado termicamente, determine o raio da esfera no momento 
do equilíbrio térmico. 
 
59 Num dia frio, as mãos são aquecidas esfregando‐se uma contra a outra.  
a) Admitir que o coeficiente de atrito entre as superfícies das mãos seja 0,5, que a força normal entre as mãos seja de 35 
N e, que a velocidade média do esfregamento de uma contra a outra seja de 35 cm/s. Qual a taxa de geração de calor? 
b) Vamos admitir, também, que a massa de cada mão seja de 350 g, que o calor específico seja da ordem de 4 kJ/kg∙K e, 
que todo o calor gerado contribua para a elevação da temperatura das mãos. Durante quanto tempo as mãos devem 
ser esfregadas para se conseguir uma elevação de 5 °C na respectiva temperatura? 
 
60  Um  bloco  de  gelo  de  1  tonelada,  destacado  de  uma  geleira,  desliza  por  um  a  encosta  de  10°  de  inclinação  com 
velocidade constante de 0,1 m/s como indica a figura. O calor latente de fusão do gelo (quantidade de calor necessária 
para  liquefação  por  unidade  de  massa)  é  de  80  cal/g.  Calcule  a  quantidade  de  gelo  que  se  derrete  por  minuto  em 
consequência do atrito. 

 
 
61 Dois cubos do mesmo material e de arestas "a" e “2a” estão nas temperaturas de 45 e 90 °C, respectivamente. Duas 
faces são postas em contato por algum tempo para atingir o equilíbrio térmico. Qual é a temperatura de equilíbrio? Não 
há mudança de fase durante o processo. 
 
62  Três  esferas  de  metal  do  mesmo  material  são  postas  em  contato.  Os  raios  das  esferas  são:  R,  2R  3R  e  estão  a 
temperaturas de 10; 10 e 20 °C, respectivamente. Calcular a temperatura de equilíbrio do sistema (em °C) 
 
63 Um calorímetro contém 50 g de água na fase líquida a 0 °C. É introduzido no calorímetro 50 g de gelo à temperatura 
de  ‐30  °C.  Determinar  a  quantidade  de  água  que  solidifica  quando  a  temperatura  atinge  o  equilíbrio,  sabendo  que  o 
calorímetro não ganhar ou perder calor. 
 

 
38 
64 Existem três barras do mesmo material, de massas m, 2m e 3m. Elas são colocadas uma sobre a outra da maneira 
indicada na figura. Sabendo que as barras central e superior estavam a temperaturas de 120 °C e 20 °C respectivamente, 
e também a temperatura inicial da barra inferior é a mesma ao final do processo, calcule a temperatura final do sistema. 

    
 
65 Em um recipiente de capacidade térmica desprezível se encontra um líquido à temperatura T1 = 10 °C. Introduz‐se 
um cubo de  metal à temperatura T2  = 200 °C no recipiente atingindo‐se uma temperatura de equilíbrio Te = 60 °C. O 
cubo  é  então  retirado  e,  quando  o  líquido  está  à  temperatura  T’1  =  20  °C,  o  cubo  é  reintroduzido,  atingindo  uma 
temperatura de equilíbrio T’e = 40 °C. Qual era a temperatura inicial do cubo neste segundo caso? 
 
66 Em um recipiente cujo equivalente em água é de 50 g são colocados 150 g de água a uma temperatura de 45 °C. Um 
bloco de gelo de 200 g de massa a ‐10 °C é colocado no recipiente. Determine a composição final do sistema. cGelo = 0,5 
cal/g °C. 
 
67  Um  calorimétrico  de  cobre  tem  uma  capacidade  térmica  de  40  cal/°C  e  contém  100  g  de  água.  O  sistema  está 
inicialmente a 0 °C. 20 g de vapor de água é introduzido dentro do calorímetro a 100 °C. Qual é a temperatura final do 
calorímetro e seu conteúdo? 
 
68 3m gramas de gelo a ‐20 °C é misturado com m gramas de vapor de água a 150 °C em um vaso de capacidade térmica 
desprezível. Qual é a temperatura final do sistema? 
 
69 De acordo com a teoria do calor específico de sólidos a temperaturas extremamente baixas (perto do zero absoluto), 
o calor específico de um sólido varia com a temperatura absoluta T como s = cT3. Onde c é uma constante que depende 
do material do sólido. Determine a energia térmica necessária para aumentar a temperatura de 0,1 kg do sólido de 0 K a 
4 K.  
 
70 Uma esfera de metal com massa de 2 kg é deixada cair a partir da altura de 160 m, e ao impactar no chão eleva‐se a 
metade  da  altura  original.  Um  quarto  da  energia  se  transformou  em  calor  e  foi  dissipado  para  o  ambiente  exterior. 
Determine o aumento da temperatura (cE = 200 J/kg °C e g = 10 m/s2) 
 
71 Um recipiente com capacidade térmica desprezável, contém 30 gramas de gelo a 0 °C. Determine a massa de vapor 
de água a 100 °C que deve ser injetado no recipiente, para se obter água líquida a 100 °C. (LF = 80 cal/g, LV = 540 cal/g) 
 
72 Um calorímetro de equivalente em água igual a 20 g, se encontra em equilíbrio térmico com 100 g de gelo e 180 g de 
água. Se 20 g de vapor é injetado a 100 °C, determine a temperatura de equilíbrio térmico e o calor transferido durante 
o processo. LF = 80 cal/g; LV = 540 cal/g. 
 
73 Em um recipiente de ferro de 40 g de massa há 50 g de gelo à temperatura de ‐30 °C. Calcular a quantidade de vapor 
de água a 150 °C que tem de ser injetado no recipiente para levar o conjunto a 20 °C. 
cGelo = 0,5 cal/g °C 
LV = 80 cal/g 
LV= 540 cal/g 
cFerro = 0,12/g °C 
cvapor de água = 0,46 cal/g °C 
 
74 Um corpo "A" de 200 g de massa está na posição mostrada na figura abaixo, 10 m acima de um calorímetro contendo 
500 cm3  de água; o equivalente em água do calorímetro é de 100 cm3. Sobre o corpo "A" colide um projétil também de 
massa igual a 200 g atingindo‐o a uma velocidade de 30 m/s. Qual é o aumento da temperatura da água quando "A" cai 
como mostrado na figura? Considere o choque perfeitamente elástico. 

 
39 
 
 
75  A  temperatura  de  massas  iguais  de  três  líquidos  diferentes  A,  B  e  C  são  12  °C,  19  °C  e  28  °C,  respectivamente.  A 
temperatura de equilíbrio quando A e B são misturados é de 16 °C, e quando B e C são misturados, é de 23 °C. Qual será 
a temperatura de equilíbrio quando A e C são misturados? 
 
76 Duas estudantes do Professor Gomes debatiam entusiasticamente sobre o processo de formação de gelo em nuvens.  
A primeira, chamada Lia, dizia: “Sabemos que a água se congela à temperatura de 0 oC, assim o gelo nas nuvens tem que 
se formar a uma temperatura próxima deste valor”. 
A  outra  aluna,  Marceli,  tinha  uma  ideia  bastante  diferente,  ela  dizia:  “Se  dividirmos  uma  quantidade  de  água  em 
pequenas  gotículas,  então  a  água  pode  super‐resfriar‐se  até  –40  oC.  Assim,  o  gelo  formado  nas  nuvens  pode  estar  a 
temperatura muito mais baixa que 0oC”.  Com qual das duas alunas você concorda? Justifique. 
 
77 Caio e Gabriel foram passar as férias em La Paz na Bolívia, que está localizada numa região cuja altitude é bem maior 
do que a de Fortaleza. O pessoal da Seleção Brasileira que o diga! Chegando lá, resolveram fazer um chá e puseram uma 
panela com água para ferver. Depois de pronto, Caio tomou um pouco do chá e notou que ele estava morno, apesar da 
água  ter  usado  água  que  acabara  de  ferver.  Gabriel  resolveu  então,  medir  a  temperatura  da  água  em  ebulição  e 
constatou que era da ordem de 85°C. Pergunta‐se:  
a) Por que razão(ões) a temperatura de ebulição da água em La Paz é menor do que em Fortaleza?  
b) Qual o procedimento de Caio e Gabriel para obter água quente a 100°C como em Fortaleza? Justificar. 
 
78 Por que a chama de uma vela está sempre voltada para cima, mesmo quando a vela não está na posição vertical? 
Explique 
 
79 A chama de uma vela sempre aponta para cima, qualquer que seja a orientação que se dê à vela. Um astronauta, em 
um  ônibus  espacial  em  órbita  da  terra,  decide  acender  uma  vela  para  ver  se  as  chamas  que  surgem  têm  o  mesmo 
comportamento do que aqui na Terra. Descreva o que você acha que deve ocorrer, comparando as duas situações. 
 
80 Duas velas, uma curta e outra longa, são acesas e colocadas no interior de um longo frasco de vidro. Quando o frasco 
é tampado, qual das velas apaga primeiro? 
 

 
a) a curta. 
b) a longa. 
c) as duas apagam no mesmo tempo. 
 

 
40 
81 Um pedaço de gelo e um termômetro mais quente são colocados num recipiente hermeticamente fechado no vácuo. 
O  gelo  e  o  termômetro  estão  suspensos  de  tal  maneira  que  não  ficam  em  contato.  Por  que  a  leitura  do  termômetro 
diminuir após algum tempo? 
 
82 Uma das extremidades de uma barra metálica isolada é mantida a 100 °C, e a outra extremidade é mantida a 0 °C por 
uma mistura de gelo e água. A barra tem 60,0 cm de comprimento e uma seção reta com área igual a 1,25 cm2. O calor 
conduzido pela barra produz a fusão de 9,0 g de gelo em 10,0 min. Ache a condutividade térmica k do metal. O calor 
latente de fusão (ou calor de fusão) do gelo é 3,35.105 J/kg. 
 
83 Um carpinteiro constrói a parede externa de uma casa usando uma camada de madeira com 3,0 cm de espessura e 
uma camada de isopor com espessura de 2,2 cm na superfície interna da parede. A madeira possui k = 0,080 W/m∙K e o 
isopor  possui  k  =  0,010  W/m∙K.  A  temperatura  da  superfície  interna  da  parede  é  igual  a  19  °C  e  a  temperatura  da 
superfície externa é igual a ‐10 °C.  
a) Qual é a temperatura na superfície da junção entre a madeira e o isopor?  
b) Qual é a taxa de transferência de calor por metro quadrado através da parede? 
 
84 Uma laje é composta por duas placas das espessuras L1 e L2 e condutividades térmicas k1 e k2 como na figura abaixo. 
As lajes têm uma área de seção transversal iguais. Encontre a condutividade térmica equivalente da laje. 

 
 
85  Na  figura  abaixo,  a  janela  é  composta  por  uma  camada  de  ar  entre  duas  placas  de  vidro  entrelaçadas  de 
condutividades térmicas kA e kV, respectivamente. Encontre a condutividade térmica equivalente do sistema. (L2 = 2L1) 

   
 
86 As três hastes mostradas na figura abaixo têm dimensões geométricas idênticas. O calor flui da extremidade quente 
a uma taxa de 10 W no arranjo (a). Encontre as taxas de fluxo de calor quando as hastes são unidas como no arranjo (b) 
e em (c). A condutividade térmica da haste A vale 200 W/m°C e das hastes B e C vale 400 W/m°C. 

 
 

 
41 
87 Tem‐se três cilindros de secções transversais iguais de cobre, latão e aço, cujos comprimentos são, respectivamente, 
46 cm, 13 cm e 12 cm. Soldam‐se os cilindros, formando o perfil em Y, indicado na figura. O extremo livre do cilindro de 
cobre é mantido a 100 °C e dos cilindros de latão e aço, a 0 °C. Supor que a superfície lateral dos cilindros esteja isolada 
termicamente. As condutividades térmicas do cobre, latão e aço valem, respectivamente, 0,92, 0,26 e 0,12, expressas 
em cal cm–1 s–1 °C–1. No regime estacionário de condução, qual a temperatura na junção? 

 
 
88  Quatro  barras  idênticas  AB,  CD,  CF  e  DE  são  unidas  como  mostrado  na  figura  a  seguir.  O  comprimento,  a  área  da 
seção transversal e a condutividade térmica de cada haste são I, A e K, respectivamente. As extremidades A, E e F são 
mantidas às temperaturas T1, T2 e T3, respectivamente. Supondo que não há perda de calor para o ambiente, encontre a 
temperatura em B. 

    
 
89  Três  hastes  feitas  do  mesmo  material  e  tendo  a  mesma  seção  transversal  foram  unidas  como  mostrado  na  figura 
abaixo.  As  hastes  tem  o  mesmo  comprimento.  As  extremidades  esquerda  e  direita  são  mantidas  a  0  °C  e  90  °C, 
respectivamente. Determine a temperatura da junção das três hastes. 

    
 
90 Duas hastes idênticas AB e CD, cada uma de comprimento L, área de seção transversal A e condutividade térmica k 
estão conectadas como mostrado na figura abaixo. Os extremos A, C e D são mantidos a temperaturas T1 = 20 °C, T2 = 30 
°C e T3 = 40 °C, respectivamente. Determine a temperatura em B. 

 
42 
    
 
91 Três hastes cilíndricas  A, B e C de comprimentos iguais e diâmetros iguais são unidas em série como mostrado na 
figura a seguir. Suas condutividades térmicas são 2k, k e 0,5k respectivamente. No estado estacionário, as temperaturas 
das  extremidades  livres  das  hastes  A  e  C  são  a  100  °C  e  0  °C,  respectivamente.  Desprezando  a  perda  de  calor  das 
superfícies curvas das hastes. 
a) Determine a temperatura da junção entre as hastes A e B. 
b) Determine a temperatura da junção entre as hastes B e C. 
c) Determine a condutibilidade térmica equivalente da combinação. 

 
 
92 Um recipiente com água foi colocado no exterior exposto ao frio até formar‐se uma camada de 5,0 cm de espessura 
de  gelo  na  sua  superfície  (figura  abaixo).  O  ar  acima  do  gelo  está  a  −10°C.  Calcule  a  taxa  de  formação  do  gelo  (em 
centímetros  por  hora)  sobre  a  superfície  inferior  da  camada  de  gelo.  Considere  a  condutividade  térmica  e  a  massa 
específica do gelo como sendo, respectivamente, 1,7 W/m.K e 0,92g/cm3. Suponha que nenhum calor flua através das 
paredes do recipiente. 

 
 
93 Um lago de água a T1 é coberto por uma camada de gelo de x1 de espessura. Se a temperatura do ar fica constante a 
T2, determine o intervalo de tempo é necessário para que a espessura do gelo aumente para x2. 
Dados: 
Densidade do gelo: ρ 
Calor latente de fusão: LF  
Área superficial: A  
Condutividade do gelo: k 
 
94  A  baixas  temperaturas  (abaixo  de  50  K),  a  condutividade  térmica  de  um  metal  é  proporcional  à  temperatura 
absoluta; isto é, k = aT, onde a é uma constante com um valor numérico que depende do material. Mostre que a taxa de 
fluxo de calor através de uma haste de comprimento L e secção transversal A, cujas extremidades estão às temperaturas 
T1 e T2, é dada por: 
a.A 2 2
 (T1  T2 )  
2L
(Ignore a perda de calor na superfície) 
 
95  Três  hastes  de  material  x  e  três  de  material  y  estão  conectadas  como  mostrado  na  figura.  Todas  as  hastes  são 
idênticas em comprimento e área de seção transversal. Se o extremo A for mantido a 60 °C e a junção E a 10 °C, calcule 
a temperatura da junção B. A condutividade térmica de x é 800W/m°C e a de y é 400W/m°C. 

 
43 
 
 
96  Três  hastes  de  um  material  x  e  três  hastes  de  um  material  y  são  conectadas  como  mostrado  na  figura.  Todas  as 
hastes são de comprimentos L e a mesma área de seção transversal A. 

 
O  extremo  A  é  mantido  em  TA  e  a  junção  E  em  TE  com  TA  >  TE.  Mostre  que  o  coeficiente  de  condutibilidade  térmica 
L(k x  3k y )
equivalente é dado por:   
Ak y (k x  k y )
 
97 Qual é a taxa de irradiação da energia por unidade de área de um corpo negro que está a uma temperatura de  
a) 273 K  
b) 2730 K? 
 
98 A emissividade do tungstênio é igual a 0,35. Uma esfera de tungstênio com raio de 1,5 cm está suspensa no interior 
de um grande recipiente a vácuo cujas paredes estão a 290 K. Que potência deve ser fornecida à esfera para manter a 
sua temperatura em 3000 K, desprezando‐se a condução de calor ao longo do suporte da esfera? 
 
99 A temperatura de operação do filamento de tungstênio de uma lâmpada incandescente é 2450 K e sua emissividade 
é igual a 0,35. Calcule a área da superfície do filamento de uma lâmpada de 150 W, supondo que toda energia elétrica 
consumida pela lâmpada seja convertida em ondas eletromagnéticas pelo filamento. (Somente uma fração do espectro 
irradiado corresponde à luz visível.) 
Respostas 
01 Devido à diferença das massas da gota e do termômetro, a temperatura de equilíbrio térmico entre eles seria muito 
próxima da temperatura em que se encontrava o termômetro. 
02 ‐12,3 °F 
03 110 °E 
04 122 °F 
05 a) Sim, pois a temperatura do paciente de 39 °C está elevada, visto que a temperatura corporal média é de 36,5 °C.  
b) Porque a temperatura do corpo dele está acima da temperatura ambiente, e sabe‐se que há uma constante troca de 
calor  entre  o  sistema  e  a  vizinhança,  onde  as  temperaturas  tendem  a  se  igualar.  Calor  é  a  transferência  de  energia 
térmica  entre  corpos  com  temperaturas  diferentes.  Como  a  temperatura  do  paciente  está  acima  da  temperatura  do 
ambiente, ele irá ceder energia térmica ao ambiente, e, dessa forma, sentirá frio. 
06 O corpo que aqueceu 1°C. Pois a variação de 1°C é maior que 1°F. 
07  a)  O  que  faz  a  coluna  de  mercúrio  subir  é  o  fato  de  a  maioria  dos  materiais  aumentarem  seu  volume  quando  se 
aumenta a temperatura. A expansão volumétrica do mercúrio o faz 'subir' pelo tubo capilar ao ocupar este espaço.  
Ocorre que é o vidro do termômetro que aquece o mercúrio, depois de ser aquecido pelo corpo. Desta forma, o vidro 
aquece  e  se  expande  antes  do  mercúrio,  aumentando  o  volume  do  reservatório,  o  que  faz  o  mercúrio  descer  para 
ocupar este espaço.  
Só então o mercúrio se aquece. De fato, o coeficiente de expansão volumar do mercúrio é muitas vezes maior que a do 
vidro, produzindo maior expansão proporcionalmente, daí ele subir! 
 
44 
b)  O  Erro  conceitual  esta  ligado  ao  conceito  de  temperatura  e  calor  pois  temperaturas  podem  ser  altas  ou  baixas  .Ja 
calor esta presente em todos os corpos em maior quantidade (Quentes ) ou em menor quantidade (Frios) 
Por  exemplo,  em  um  dia  quente,  usa‐se  a  expressão  “Hoje  está  calor!”.  Porém,  corpos  com  baixas  temperaturas 
também possuem calor, só que em menor quantidade. Isso quer dizer apenas que a agitação das moléculas é menor em 
corpos “frios”. 
c)  Falando  do  vácuo  absoluto,  não  existe  matéria.  Portanto  não  existem  moléculas  vibrando.  Não  há  como  medir  a 
temperatura pois qualquer objeto colocado lá irá emitir radiação até entrar em equilíbrio. 
08 B = (18A + 216)/5 
09 C(x) =  x²/60 + 2x/3 
10 P' = 444 mm de mercúrio  
11 0,964 atm 
12 191 °F 
13 360 ° 
14 10.10–5  °F‐1 
15 Sofre um deslocamento de 1,8 mm para a direita e 1,8 mm para baixo 
  2B
16  barra  A  
3
17 2,3.10‐5 °C‐1 
18 66 °C 
x 2
19    
y 3
20 a) αI = 1,0 ∙ 10–5 °C–1; αII = 2,0 ∙ 10–5 °C–1; b) metal II 
21 120 °C 
22 100,8 m 
23 γ = αx + αy + αz 
4   b
24   x  a  
5
25 10 cm 
1
26    (1  2 )  
2
1 1
27    
2 4
28 Diminuirá 
  1  1 t 
29  n  1   
2  1  2 t 
30 26 s 
31 51,84 s 
32 9,2 °C 
33 a) Atrasa         b) 8 s por dia 
34 2,5.10‐5 °C‐1 
L
35  t  2 1  X 1/2  
g
36 a) aumento de 0,36%.    b) 0,18%    c) aumento percentual de 0,0054%. 
d) a massa não modifica com a temperatura.      e) 0,0018 °C‐1  
37 0,1 cm 
38 3 % 
39 a) 36%        b) irá derramar 
40 210 °C 
41 R/r = 2 
42  Isto ocorre porque a capacidade calorífica da areia é muito pequena, assim apesar do sol intenso durante o dia, o 
calor armazenado pela areia neste período é rapidamente transferido para o ar e as noites, tornam‐se muito frias no 
deserto. 
43 Concluímos , portanto, que a chave seletora se encontrava na posição “inverno”. 
44 a) 419000 esferas      b) 102 
45 360 m/s 
 
45 
46 50 g 
47 0,31 ml/s 
48 75 g 
49 i) C' = 50 J/°C; ii) C' = 0,1 J/°C   b) No segundo caso 
50 0,07 °C 
51 a) convecção      b) 1,4 kWh 
52 a) 150.106 toneladas de gelo   b) 555 m 
53–48,6 °C 
54 21 min 
55 As cores que o projétil apresentou são verde, azul e violeta. 
56 a) 0 °C        b) 2,51 °C 
57 25,3 kg 
  m .c .(T  T )  
58  R  r0 . 1  .  2 2 2 1    
  m1 .c1  m2 .c2  
59 19 min 
60 30,5 g 
61 85 °C 
62 17,5 °C 
63 9,375 g 
64 60 °C 
65 96 °C 
66 O sistema está a 0 °C contendo 250 g de água e 100 g de gelo. 
67 80 °C 
68 98,5 °C 
69 6,4c Joule 
70 3 °C 
71 10 g 
72 T = 15 °C; Q = 12800 cal 
73 9,32 g 
74 Δt = 0,053 °C 
75 20,3 °C 
76 Marceli está correta 
77  a) O ponto de ebulição da água depende da pressão atmosférica a que está submetida. Como a altitude de La Paz é 
bem maior que a de São Paulo, sua pressão atmosférica é menor. Com uma pressão atmosférica menor diminui também 
a temperatura de ebulição da água. Por isso, Gabriel verificou que, em La Paz, a água fervia a aproximadamente 85 °C.  
b) Para se elevar a temperatura de ebulição da água, deve‐se aumentar a pressão a que ela está submetida. Isso pode 
ser obtido utilizando‐se, por exemplo, uma panela de pressão. A panela de pressão é projetada para ferver água acima 
do  ponto  de  ebulição  normal  devido  ao  aumento  da  "pressão  absoluta"  no  interior  da  panela,  que  é  obtido  pelo 
controle da quantidade de vapor feito pela "válvula de escape". Em cima do "furinho" da válvula existe uma massa "m" 
dimensionada para que, sempre que a pressão absoluta do vapor atingir um valor pré‐fixado, ela é erguida, deixando 
escapar vapor, controlando assim a pressão absoluta no interior da panela. Com o devido cuidado, pode‐se obter água a 
temperaturas acima do ponto de ebulição (em São Paulo, por exemplo, pode‐se obter água a temperaturas maiores que 
100 °C). Utilizando‐se uma panela de pressão pode‐se obter água quente a 96 °C em La Paz. 
78  A chama fica sempre para cima porque, durante a queima da vela, ocorre um processo contínuo em que o produto 
da combustão sai pelo topo da chama, enquanto mais oxigênio entra pela sua base, próximo de onde a parafina está 
sendo queimada. Qualquer que seja a posição da vela, o fogo sempre se mantém no sentido vertical. O ar frio é atraído 
para a chama por causa das chamadas correntes de convecção. Como os gases produzidos pela combustão são menos 
densos que o ar ao redor, eles sofrem menor ação da força da gravidade e sobem, ao contrário do ar frio, atraído pela 
gravidade da Terra por ser mais denso (mais pesado). 
O ar quente, ao subir, deixa espaço para que o ar frio, com mais oxigênio, ocupe o seu lugar, realimentando o processo 
de combustão da chama. A fuligem, produto da queima incompleta da vela, também é carregada para cima durante a 
combustão. Portanto, a convecção do ar ocorre graças ao efeito da gravidade. Sem ela, não há o movimento constante 
do ar frio, carregando oxigênio para dentro da vela. 
79 Em um ambiente onde não haja peso não haverá convecção. O ar aquecido, cujo oxigênio foi consumido pela chama, 
permanece próximo ao pavio, e não havendo oxigênio para a combustão, a chama se apaga. 
80 A vela longa apaga primeiro. 
 
46 
81 O termômetro transfere calor por irradiação. 
82 201 W/m.K 
83 a) 5,8 °C    b) 11 W/m2 
k k (L  L )
84  k e  1 2 1 2  
k1L 2  k2L1
2k k
85  k e  A v  
kA  kv
86 400 W 
87 40 °C 
3T  2(T2  T3 )
88  T  1  
7
89 60 °C 
90 32 °C 
2k
91 a) 85,7 °C    b) 57,1 °C    c)  k eq   
7
92 0,40 cm/h 
L..(x 22  x12 )
93  t   
2k.T
94 Demonstração 
95 40 °C 
96 Demonstração 
97 a) 315 W/m2    
b) 3,15.106 W/m2 
98 4,54.104 W 
99 2,1 cm2 
 
 
 
 
 
 
 
 
 
 
 
 
 
 
 
 
 
 
 
 
 
 
 
 
 
 
 
 
 
 
 

 
47 

Potrebbero piacerti anche